Этого треда уже нет.
Это копия, сохраненная 28 июля 2019 года.

Скачать тред: только с превью, с превью и прикрепленными файлами.
Второй вариант может долго скачиваться. Файлы будут только в живых или недавно утонувших тредах. Подробнее

Если вам полезен архив М.Двача, пожертвуйте на оплату сервера.
Тред тупых вопросов №103 EINSTEIN EDITION 464488 В конец треда | Веб
Тред вопросов о жизни, Вселенной и всём таком.

Спрашиваем то, за что в других местах выдают путёвку в биореактор. Здесь анонимные учёные мирового уровня критически рассмотрят любые гениальные идеи и нарисованные в Paint схемы.

Предыдущий тут: >>459163 (OP)
https://2ch.hk/spc/res/459163.html (М)

Q: Можно быстрее?
A: Можно упасть в пузырь альбукерке, наса уже почти надула его.

Q: Я начитался охуительных историй про уфологию, че делать, нам жопа?
A: Да, тебе жопа, можешь сгонять в зогач или куда оттуда пошлют.

Q: Что будет с человеком в вакууме без скафандра / если он упадет на черную дыру / попробует ступить на поверхность газового гиганта/солнца?
A: Он умрёт.

Q: Почему бы не привязать ракету к воздушному шару или стартовать с горы?
A: Космос - это не как высоко, а как быстро, большая часть энергии ракеты уходит на разгон вбок.
Подробнее тут https://what-if.xkcd.com/58/ (английский) https://chtoes.li/orbital-speed/ (перевод)
Einstein19211.png131 Кб, 486x605
2 464490
>>64488 (OP)
Ебанный стыд...
Во-первых, Алькубьерре.
Во-вторых, не упасть, а создавать вокруг корабля изнутри (иначе кина не будет).
В-третьих, НАСА искривляет пространство на десятимиллионную часть, контролируя это сверхточными интерферометрами, до самого варп-привода здесь - как до Антарктиды раком.
3 464491
>>464487
Подожди, так разве это не обычная теория гравитации с гравитонами, просто с заменой знака на противоположный? Этакое море Дирака, гравитация эдишен.

Ну, как если бы мы рассуждали об электромагнитном взаимодействии, просто вместо электрона как базовой частицы взяли бы дырку — его отсутствие. Тогда бы, например, при рассмотрении атома у нас бы вместо окружающего вакуума получилась заполненная дырками внешняя среда, а атом вместо электронных облаков имел бы поля разрежения. И так далее.

Тогда, конечно, все предсказания будут выполняться и все уравнения совпадут один в один, просто непонятно, а в чем смысл такой теории?
4 464492
>>64488 (OP)

>WarpedSpaceandT[...].webm


Это что при слиянии черных дыр образуется антигравитация?
5 464493
>>64491
В том-то и дело, что "альтернативы" ОТО либо оказываются ОТО in disguise, либо проебываются по воспроизводству ее предсказаний.
6 464494
>>64491

>Подожди, так разве это не обычная теория гравитации с гравитонами, просто с заменой знака на противоположный?



Нет, как до этого анон написал почти чисто механистическая теория.

И это не гравитоны, мне само название не нравится, ведь оно ассоциируется с откровенной глупостью - кинул шарик в груду других шариков, а они с какого-то хрена притянулись, а не получили импульс.

Здесь идея именно в том, что есть так называемый первород, из которого состоят протон и протон со свёрнутым полем.
Электрон это присоединённый вихрь из газовой динамики. То, что мы наблюдаем это грубо говоря осколки, ну или группа, большая учитывая их размеры, которая обладая изначальными скоростями собралась такой же, но гораздо менее долго живущий вихрь. По предположениям протоны живут около 10 миллиардов лет, дальше распадаются. То, что в центре галактики создаёт протоны новые, оно по теории может быть вечно, если ещё одно такое не объявится рядом.

>в чем смысл такой теории?



В том, что сразу становится ясно как создать двигатель, который позволит летать к звёздам - по этой теории, внутри тел с гравитационным полем крайне слабо или даже совсем не действуют внешние силы гравитации и сама инерция, ведь её природа кроется в самой сути этой среды, получается такой же эффект, который нам показывают изображая ЧД, окружающее пространство как бы обтекает такой вихрь и слабо мешает движению. То есть если резко остановить Землю, люди с неё не улетят, они даже не заметят этого. Теперь к движку, тоже самое, любые ускорения не будут влиять на тех кто внутри, получается за одну секунду можно будет долететь до плутона или до другого края галактики и даже в другую галактику.
7 464495
>>64494
Для мелких частиц должны быть охуенно сильные квантовые эффекты.

>протоны живут около 10 миллиардов лет, дальше распадаются


Что несешь, вообще ахуеть.

>если резко остановить Землю, люди с неё не улетят, они даже не заметят этого


ЧЕЕЕЕЕЕЕЕЕЕЕЕЕЕЕЕЛ!
8 464496
>>64495

>>если резко остановить Землю, люди с неё не улетят, они даже не заметят этого


>ЧЕЕЕЕЕЕЕЕЕЕЕЕЕЕЕЕЛ!


что не так блять?
sage 9 464497
>>64496

>30 км/с орбитального движения


>резко остановить


>не улетят

10 464498
>>64494
>>64495
>>64496
>>64497
Два дебила - это сила.
11 464501
>>64495

>Для мелких частиц должны быть охуенно сильные квантовые эффекты.



Они и должны наблюдаться, ведь для них влияние этой среды будет сильнее.

Касаемо Земли, опять же выше я привёл объяснение уровня 17 века, ну может 18.
12 464504
>>64488 (OP)
С какой высоты начинается космос?
Есть ли чёткая граница атмосферы?
На какой предельной высоте удерживаются гравитацией атомы воздуха?
13 464507
>>464486

>Вся суть механистического объяснения как раз в экранировании телами друг друга от давления внешней среды.


Если клепать механистичную теорию гравитации, то можно отождествить ее с температурой в "гравитонном" газе, типа холодные тела приталкиваются в горячем газе. Экранирования при этом нет, просто градиенты температур вокруг тел накладываются друг на друга.

>Эксперименты по измерению скорости гравитационных волн показывают, что они распространяются со скоростью света, так что тоже не катит.


Скорее всего это не точные эксперименты. Механическую теорию гравитации невозможно построить без привлечения сверхсветовых скоростей, что дает основание в сомнении к таким результатам экспериментов. Да и гравитационные волны исходя из такого подхода представляют собой волны температуры, которые очень быстро затухают в среде, то есть в эксперименте возможно измеряли не скорость гравитационных волн, а нечто другое.
14 464508
>>64504
100 км
не очень
высоко, вплоть до тысяч км
15 464509
>>64488 (OP)

> пик 2


Почему именно четыре ярких источника, а не смазанное кольцо?
16 464512
>>64508
Спсбо, ну т.е. не один атом воздуха не улетит от нас.
А то я чёт переживал, что через озоновые дыры всё
сдрыснет в холодный вакуум и подзадохнёмся тута.
17 464514
>>64507

>Скорее всего это не точные эксперименты. Механическую теорию гравитации невозможно построить без привлечения сверхсветовых скоростей, что дает основание в сомнении к таким результатам экспериментов.



Да просто механистическая теория говно нерабочее, вот и все.

>то есть в эксперименте возможно измеряли не скорость гравитационных волн, а нечто другое.



https://ru.wikipedia.org/wiki/GW170817 — вот это наблюдение, про которое я говорил. В течение 2 секунд в одной области неба произошел гамма-всплеск и из того же самого места был зафиксирован приход гравитационных волн, при этом соответствующий по амплитуде наблюдениям. Как-то уж совсем маловероятно для совпадения.

Сверхточные интерферометры, те самые из копипасты, меряют изменение метрики пространства, сложно представить, что на их показания могло повлиять что-то помимо гравитационных волн.
18 464521
>>64512
Ну, в общем так и есть, Земляшка постепенно теряет атмосферу, но до превращения Солнца в красного гиганта нам ее скорее всего хватит.
19 464522
>>64514
Нет, сама идея изменения расстояния в некоторых системах говно нерабочее и вызывает только смех. То что там они измеряют может быть что угодно.

Касаемо наблюдения. Амплитуда чего? Гамма излучения, которая подобна или равна гравитационной?
20 464523
>>64521

>Земляшка постепенно теряет атмосферу


Да, но при этом что-то попердывают вулканы, то есть выходят газики, растворенные в мантии. Какой там баланс выходит - хз. К тому же, теряются в основном легкие водород и гелий, которые один хуй никому не нужны.
21 464524
>>64512
Воздух держится у нас гравитацией, и ещё магнитное поле планеты защищает от того, чтобы его солнечным ветром не сдуло. Озоновые дыры это немного другое, озон это тоже газ, он просто солнечный ультрафиолет до поверхности не пропускает.

Но атмосфера всё равно потихоньку теряется, однако так медленно, что нам нечего бояться. Тебе на жизнь хватит.
sage 22 464525
>>64522
Иди нахуй, говно.
23 464526
>>64525

Ожидаемо, но ты не тот анон, что мне отвечал на вопросы, ты тупее, если тебе настолько неприятна эта тема, то игнорируй. Напоминаю, это тред тупых вопросов. Или тут уже успели появиться запретные темы? Ясное дело, что отвечать на вопросы уровня а чё ето атомы не улетаю с Земли проще, чем разъяснить в чём я не прав.
24 464548
>>64508

>100 км


Скоро на 80 км опустят - для НАСА и ВВС США - космос уже давно начинается на 80 км.
Фактически космос начинается там - где воздух не оказывает существенного влияния на орбиту, то есть примерно с 80 км.
Сам Карман установил границу на высоте 83,9 км - но для него это была высота, куда может долететь самолет - используя подъемную силу.
Есть ещё и военная составляющая - ибо космос общий, а воздушное пространство до космоса принадлежит отдельным странам. Чем выше космос - тем спокойнее.
https://www.fai.org/news/statement-about-karman-line
sage 25 464583
>>64526
Ну что ж ты так. За добавкой вернулса маня?
26 464656
>>64583

>Мам, пространство может искривляться и похуй, что это пустота


>Мам, быстрее света ничего двигаться не может и похуй, что это выведено из преобразований Лоренца



Вот тебе добавки. Странно, что вообще сейчас есть люди, что всерьёз в эти бредни верят, именно верят, ведь здравый смысл это отметает.

Ну и напоследок, почитай, что про все эти бредни писал Циолковский.
27 464667
Чем запуск отличается от пуска?
28 464716
>>64667
Тем же, чем заебенить отличается от ебенить.
sage 29 464734
>>64656

>здравый смысл это отметает


>что про все эти бредни писал Циолковский


Уходи. Нахуй.
30 464736
>>64656
А где на дваче можно пообсуждать эфир в кругу единомышленников, без тролльских эйнштенодрочеров?
31 464761
>>64736
/fs
/mg
/zog
32 464777
>>64736
Отдельный тред создать, если действительно много есть что пообсуждать, а если дискаса мало, то можно прямо тут.
sage 33 464778
>>64777

>заговорчики в /zog/

34 464782
>>64778
https://wikilivres.org/wiki/Äther_und_Relativitätstheorie

>Nach der allgemeinen Relativitätstheorie ist der Raum mit physikalischen Qualitäten ausgestattet; es existiert also in diesem Sinne ein Äther. Gemäß der allgemeinen Relativitätstheorie ist ein Raum ohne Äther undenkbar; denn in einem solchen gäbe es nicht nur keine Lichtfortpflanzung, sondern auch keine Existenzmöglichkeit von Maßstäben und Uhren, also auch keine räumlich-zeitlichen Entfernungen im Sinne der Physik.



Перевод:
Общая теория относительности наделяет пространство физическими свойствами; таким образом, в этом смысле эфир существует. Согласно общей теории относительности, пространство немыслимо без эфира; поскольку в таком пространстве не только было бы невозможно распространение света, но и не могли бы существовать масштабы и часы и не было бы никаких пространственно-временных расстояний в физическом смысле слова.

Даже Эйнштейн не против был эфира, говорил что его теория эфирная, просто модель эфира несколько иная. Так почему бы не обсуждать остальные эфирные модели если эфир есть?
35 464787
>>64782

>Так почему бы не обсуждать остальные эфирные модели если эфир есть?


Наконец-то шизик созрел для Тупого Вопроса. Ответ на него очевиден остальные эфирные модели надежно опровергаются результатами наблюдений и экспериментов, но шизик продолжает делать вид, что 100+ лет релятивизма — это на самом деле в говне моченые все подстроили, ага. С этим лучше в /zog.
36 464788
Предположим, что мы покрываем Меркурий тонким слоем вещества с очень высокой отражательной способностью и по-тихоньку ждём, пока его орбита растянется до орбиты Венеры. Насколько сложно будет сделать его спутником Венеры? Сколько придётся ждать и сработает ли это вообще? Есть ли какие-нибудь более эффективные способы?
37 464793
>>64788
С чего ты взял, что это вообще будет иметь какой-то эффект? Максимум - это увеличение импульса от светового давления в два раза (полное отражение по сравнению с полным поглощением).
38 464794
>>64793
Ну не ракетами же его двигать. Первое что пришло в голову из околоосуществимого.
39 464797
>>64794
Если бы Солнце еще светило Меркурию в жопу, а не в бок по направление движения (что, понятно, невозможно в системе, где Меркурий вращается вокруг Солнца), что-то может и вышло бы. Хотя подозреваю, что это световое давление настолько ничтожно, что и в этом случае ничего бы не дало.
40 464798
>>64794
Значит, укрепляем поверхность, ставим огроменное пихло и потихоньку начинаем ускорять планету выбирая нужные моменты для запуска. Я, правда, даже хуй знает что сложнее, построить и притащить топливо для такой йобы или обмазать весь шарик блестяшками.
4e3f655a6fc421.png538 Кб, 854x484
41 464799
>>64488 (OP)
Я правильно понимаю, что в этих половых губах на скриншоте из вебм кратковременно возникает отрицательная кривизна пространства, и скорость света там больше обычной С?
42 464804
>>64799
Почему ты думаешь, что она там больше с?
43 464809
>>64787
Из чего состоит бозон и из чего состоит это пространство? М?
44 464812
Сап, спейсаны. Возможно вопрос не совсем по адресу, но всё же.
Собственно, наука уже выяснила почему скорость света именно такая какая она есть а не другая?
45 464814
>>64787

>остальные эфирные модели надежно опровергаются результатами наблюдений и экспериментов


Слишком самоуверенное заявление.
46 464815
>>64804
Потому что там должно происходить нечто обратное гравитационному замедлению времени. Если провал подразумевает гравитационную яму с замедлением времени, а бездна под горизонтом событий - область, где время с точки зрения внешней вселенной останавливается, то в пучности время должно идти быстрее внешней вселенной. Ну и местная скорость света должна быть больше С.
47 464816
>>64812
Не.
Наука даже до сих пор не выяснила, почему число 8 именно такое, а не другое. Не говоря уж о всяких там числах пи...
48 464819
>>64488 (OP)
Как думаете может России отказаться вообще от космических программ, все равно нихуя не выходит, а так хоть деньги лишние будут.
49 464821
>>64816
Сейчас бы в 2к19 не понимать разницы между математическими абстракциями (8 и пи) и физическими константами (скорость света).
50 464824
>>64819

> а так хоть деньги лишние будут.


Разворуют.
51 464829
>>64812

>Собственно, наука уже выяснила почему скорость света именно такая какая она есть а не другая?


Конечно, скорость света ограниченна временем взаимодействия частиц эфира при передаче импульса, который ошибочно называют фотоном, на основании того факта, что конечная частица при передачи импульса через эфир выступает в роли фотона.
52 464832
>>64815

>должно


Ясно
Дальше этого дебила не читал.
53 464833
>>64812
В естественной системе единиц скорость света равна единице, так как у фотона нулевая масса покоя. Ты неправильно формулируешь вопрос. Правильный вопрос - не почему свет движется с такой скоростью, а почему у других частиц есть массы покоя, не позволяющие им двигаться с единичной скоростью. Квантовая теория поля объясняет массы частиц (кроме нейтрино) через взаимодействие с бозоном Хиггса. Однако почему они взаимодействуют с ним именно так, а не иначе - сия тайна велика есть.
54 464835
>>64829

>этот протекший из обо/сцайной параши малолетний шизодебил


Понятно.
Dirac.jpg34 Кб, 450x610
55 464837
>>64835
Почему обосцай скатился в такое говно?
56 464839
>>64837
Всегда был парашей для говна и шизиков, со времен закрепленного треда где есть мой пост, хаха уже полностью.
57 464842
>>64839
Вопрос по теме закрепленного треда: хуи сосешь?
58 464844
>>64842
Мод-тян, ты?
card 027 Jennifer Ouellette.png618 Кб, 539x800
59 464850
>>64844
Мод обосцача - тян?
60 464853
>>64850
Сейчас уже в любом случае нет.
Теперь в моче остались только школодебилы из самых червепидорских здесь тоже, кстати
61 464857
>>64853
Мочу вроде несколько раз выселяли же отсюда и сразу начинался вайп говном и спам антисетипетухами?
62 464874
>>64832
Если ты такой умный, объясни! Тебе сложна что ли?
63 464880
>>64833
Давай проще, на перестроение частиц эфира требуется некоторое время, поэтому импульс через плотноприжатые сферообразные частицы эфира не может распространяться мгновенно.
64 464893
>>64874
Деньги вперед.
>>64880
Да ты, холоп, не уймешься.
65 464894
>>64893

>Деньги вперед.



В светоносную среду веруешь?
66 464897
>>64894
Где деньги, маня?
67 464898
>>64897
Их нет, да я и сам разберусь, безплатно.

Время в этих участках почти безконечно ускорилось, из чего следует, что либо два этих шарика помереть должны были, испариться, либо потерять как минимум половину массы, пока это ускорение не прошло.

Вывод: слияние двух и более ЧД может привести к их полному испарению.
68 464901
>>64898

>Их нет


Дальше читать нищего шизодебила смысла нет.
69 464924
>>64894

>В светоносную среду веруешь?


а что лучше в мистические поля веровать, которые как бы есть, но которых и нет. это же уровень детсада.
а наличие светоносной среды в виде плотноприжатых сфер многое объясняет. например инерцию, когда сферы с фронта движения перетекают в тыл и тем самым поддерживают движение тела и тд.
70 464932
>>64924
про гравитационные волны что расскажешь?
71 464933
>>64901
О-объективность.

А зачем ты тогда нужен, если по факту ничего объяснить не способен? Да, вот они, учёные 21 века!

>>64924
Чем тебе прост газ не угодил, то? Тут выше анон отличную идею про быстрое затухание тепловых волн предложил, ща изучать буду.
Про инерцию согласен, что эта самая среда её и создаёт и поддерживает.
72 464935
>>64932

>про гравитационные волны что расскажешь?


эфирная среда и порождает гравитацию, а волна в среде это типичное явление
>>64933

>Чем тебе прост газ не угодил, то?


пустота невозможна, размерностью может обладать только материальное тело. соответственно всё пространство заполнено материей без образования пустоты. но такое пространство будет подобно бетону без возможности к движению. однако частицы материи без осевого вращения могут свободно делиться и объединяться, образуя тем самым идеальную жидкость, которая оптимальным образом подстраивается под промежутки между плотноприжатыми сферами, не допуская образования пустоты. солнце является буфером идеальной жидкости в центре вихря из плотноприжатых сфер.
73 464937
>>64933
>>64935
Два дебила - это сила.
Обо/сцай на выезде, хаха.
А разгадка проста: весна, шизики обостряются
74 464941
>>64935
Эта среда как жидкость уже не раз терпела провал, беда в этом. Там вихри или не могут исчезнуть или не могут образоваться.

>>64937
Если ты действительно учёный, то это печально, ведь твоя задача не обезьянничать, а указать на ошибку.
75 464942
>>64941

>указать на ошибку.


Очевидно же что ошибка у тебя в количестве хромосом
76 464943
>>64942
Жаль, что ты не способен на конструктивную критику, поэтому то и у нас потешная космическая эра.
77 464944
>>64943

>поэтому то и


>потешная


Оно еще и деревенское.
Впрочем, ничего нового.
78 464945
>>64944

Хех, знаешь, ты мне напомнил одного долбоёба, что доказывал так называемую закопанность домов - звёздной пылью, я не шучу, это оно дейсвтительно изрыгало из своего рта, выбросом мусора и его не уборкой, в деревнях. Этот долбоёб учёный типа кстати и так же обо всех неумных отзывался.

Эх, благодарю тебя за чёткое понимание полного и окончательного краха как системы образования, так и науки.
79 464946
>>64945

>дейсвтительно


А ручонки-то дрожат.

>выбросом мусора и его не уборкой, в деревнях.


>эта ОБИДА копротивляющегося деревенского быдла


Ясно.
80 464947
>>64945
Ну если разбираться, то так оно и есть. Все химические элементы кроме H это звёздная пыль, по сути.
81 464948
>>64946
С чего ты взял, что я на тебя обиделся? Я прост жду, когда ты мне ответишь, хотя бы на невозможность образования тороидального вихря в газе с более меньшим трением частиц, вязкостью и так далее.

>>64947
Это понятно, но объяснять подобным образом данное явление это заявка на безумие.
82 464950
>>64941

>Эта среда как жидкость уже не раз терпела провал, беда в этом. Там вихри или не могут исчезнуть или не могут образоваться.


хрень, такая теория даже никогда и не обсуждалась, хотя и спизжена у декарта.
вихри образуются из плотноприжатых сфер, идеальная жидкость тупо заполняет промежутки. правда когда идеальная жидкость теряет скорость, то она превращается в частицу и если эта частица приобретет осевое вращение (спин), то она не будет разрушена. так рождаются всякие спиралевидные и прочие элементарные частицы.
83 464951
>>64948

>тороидального вихря в газе


ацюковский это же шарлатан, что у него находится в промежутках между частицами газа? пустота? но она невозможна! а если всё пространство заполнить частицами газа без образования пустоты, то мы получим бетон в котором движение невозможно.
84 464952
>>64951

>что у него находится в промежутках между частицами газа



Это такое же предположение, как и постоянство скорости света, ничего более, и потом уже от этого шагаю остальные предположения. Что между ними сложно ответить.

Моё мнение, что её нужно проверить, я размышляю над этим.
85 464954
>>64948

>эти оправдания деревенского омегана


Ясно.
>>64950
>>64951

>эти беседы с залетным обо/сцайным шизиком


Понятно.
>>64952

>я размышляю


Лучшая шутка месяца.
86 464956
>>64954
Почему ты такой злой и агрессивный? Тебя кто-то обидел?
87 464957
>>64952

>Это такое же предположение, как и постоянство скорости света, ничего более, и потом уже от этого шагаю остальные предположения.


а вот это главная блядь ошибка. вы сперва логичную модель эфира постройте, а потом шагайте дальше. а то хренота получается: "мы сперва допустим существование бреда, а потом от него шагать будем"
надеюсь, ты понимаешь, что газами никогда пространство на 100% не заполнить, иначе это будут уже не газы, а пустота невозможна. ну не может быть пустого пространства. как "ничто" может обладать размерностью?
88 464958
>>64956

>эти риторические вопросы


Верный признак и симптом лоха по жизни, кстати.
89 464959
>>64957

>этот мелкобуквенный шизик


Они на свет лезут.
90 464960
>>64958

>я обтекаю


Сливаться ты умеешь, пиздуй по холодку.
91 464961
>>64960

>эта ОБИДА НА ПРАВДУ


As expected.
92 464962
>>64957

Забыл, утверждается, что пространство безконечно и так же газ его безконечно заполняет. Знаю, нарушение закона сохранения энергии, но изначально твоё требование удивляет, ведь и та же теория большого взрыва имеет подобного рода проблемы.
93 464963
>>64962

>безконечно


>безконечно


Ох уж это деревенское быдло.
94 464965
>>64963
Стопе, ты с антропогеза что ли? Я тебя задел? Да? Ну что поделать, не нравитесь вы мне, я бы ни не ругался, если бы не подход к доказательствам и опровержениям, которые даже хуже чем это могут сделать пятиклашки.
95 464966
>>64965

>?


>?


>?


Ебать ты чмо омежное по жизни.
96 464967
>>64962

>Забыл, утверждается, что пространство безконечно и так же газ его безконечно заполняет


как может существовать газ без образования пустоты? ацюковский шарлатан, че его слушать.
я знаю его бредни между крупным газом более мелкий газ и так до бесконечности, но вся правда кроется в том, что для того чтобы пододвинуть частицу самого крупного газа, нам нужно бесконечно много перестроений более мелких, а это займет бесконечно много времени.
опять же из той теории ацюковского следует, что он допускает пустоту в бесконечности, а если её нет, то у него бетон, а не газы.

>ведь и та же теория большого взрыва имеет подобного рода проблемы


а это вообще бредовая гипотеза!
материя не рождается и не исчезает - она переходит из одной формы в другую. ошибка даунов, которые выдумали ТБВ в том, что у них энергия может существовать отдельно от материи, а это невозможно! энергия это мера движения материи, энергия без материи невозможна.
97 464968
>>64966
То что меня тут забанят это был вопрос времени, смешно, ты действуешь как омежка и бежишь жаловаться маме, вместо того чтобы ответить по делу. И при этом ты первым стал на меня нарываться и совершенно игнорировать мои вопросы, только оскорбления в ответ.

Зарепортил в ответ за щитпостинг.
98 464969
>>64968

> И при этом ты первым стал на меня нарываться и совершенно игнорировать мои вопросы, только оскорбления в ответ


а ему нечего ответить. в официальной физике такие бредовые теоретические выкладки, что они не выдерживают никакой критики. если бы он с нами вступил в спор, то это было бы аналогично томы, что он бы снял штаны и нагнулся.
99 464970
>>64968

>этот спалившийся дегенерат с обо/сцая


Надо же, эта гнида и правда оттуда, хаха.

>ответить по делу


Тут как нельзя более уместны золотые, бессмертны слова: соси хуй, быдло
100 464971
>>64969

>он с нами вступил в спор


Споры - они в реальной ИРЛ жизни, хикка мамина.
Но там ты молчишь и глазки прячешь, хаха.
101 464972
>>64967
Так и с жидкостью и с полями всё та же проблема, откуда оно взялось вообще и из чего состоит.

Любопытно, что мы снова приходим к дифференциалам и интегралам, а именно к безконечному приращению. Думаю, что такой вариант с безконечно вложенными частицами возможен, просто с каждым новым уровнем так сказать, скорости всё выше и выше, тогда и никаких проблем с задержками не будет.

Твоё объяснение меня заинтересовало. Вопрос в том, откуда то, о чём ты пишешь.
102 464973
>>64970
Бля, лучше уж тут сри, не мешай тем рыбятам, там интересная тема, причём без стремлений нарушить ваши догмы, я лишь некие мысли выложил, не более.

Какой же ты додик.
103 464974
>>64973

>рыбятам


>додик


Деревня, деревня never change.
И нищий к тому же, хехе.
104 464977
>>64972

>Так и с жидкостью и с полями всё та же проблема, откуда оно взялось вообще и из чего состоит.


первоначально была единая протяженная материя, под действие вложенного движения она мгновенно разделилась на части. поскольку пустоты не существует, то частицы не могут покинуть первоначальный объём, но они обладают движением. единственный выход для них это наяривать по кругу, образуя солнечные системы. наяривая по кругу они стачиваются до идеальных шариков с осевым вращением, а осколки без осевого вращения заполняют промежутки, бесконечно делятся и объединяются, образуя идеальную жидкость.
спин, осевое вращение, защищает частицу от объединения с другими.

>Думаю, что такой вариант с безконечно вложенными частицами возможен, просто с каждым новым уровнем так сказать, скорости всё выше и выше, тогда и никаких проблем с задержками не будет.


думаю что нет, потому что само признание бесконечности подразумевает, что где-то там в бесконечности существует пустота, которая невозможна.

>Твоё объяснение меня заинтересовало. Вопрос в том, откуда то, о чём ты пишешь.


декарт, "первоначала философии", "мир или трактат о свете"
только у первого писания перевода на русский нет, забанили за правду. но благо немцы молодцы - http://www.zeno.org/Philosophie/M/Descartes,+René/Prinzipien+der+Philosophie/3.+Von+der+sichtbaren+Welt
7lbydds5b4t11.jpg64 Кб, 720x481
105 464978
>>64977
пикчу забыл)
106 464980
>>64977

>пустота, которая невозможна


Аристотель, ты? охуевший дурачок тот еще был, кстати
107 464981
>>64977

Интересная идея, почитаю. Сам сторонник того, что всё таки из небытия так сказать появилось, так что первопричина есть точно.
108 464982
>>64980

>Аристотель, ты?


размерность это свойство, а свойством может обладать только материальное тело. "ничто" не может обладать свойством, потому что его нет, оно не существует.
пустота родилась из-за невежества людского, раз не вижу, то нет. а как можно увидеть носитель света? как можно увидеть среду, через которые распространяются импульсы, которые мы зовем светом, где конечная частица в цепочке выступает в роли фотона. как можно увидеть частицы, образующие идеальную жидкость и заполняющие промежутки между сферами? тупые человечки.
но вообще под пустотой подразумевается вакуум, то есть отсутствие молекул. эм волны то продолжают распространяться, а значит есть среда для волн.
109 464983
>>64982

>эта мелкобуквенная шизофазия


Весна идет, весне дорогу :3
110 464988
>>64981

>Интересная идея, почитаю.


у декарта хорошая модель эфира, рабочая. но всё же на некоторых этапах следует делать поправку на 17 век. не все процессы он верно понимает, отсюда не всё верно объясняет. электричество тогда было вообще на уровне статики.
но главное иметь саму модель эфира - это основа. а понять процессы не составит труда.
вот например трехгранные спиралевидные частицы декарта очень похожи на нейтрино и тд.
111 464989
>>64983

>я нихуя не понимаю, и ответит мне нечем, но я мамкну, что я тут


давай, давай, пиздуй
изображение.png287 Кб, 360x540
112 464990
>>64989

>я мамкну


Кто о чем - а школьник о мамках.
1511604513135756285.jpg357 Кб, 1440x1800
113 464991
>>64990

>школьник


действительно, школьнику везде школьники мерещатся
114 464992
>>64991
Зарепортил аватарку.
115 464993
>>64990
репорт
116 464994
>>64993

>этот обиженный школошизик


Ясно.
117 464995
>>64994

>я невнимашка и школоеб, я нихуя не могу вам ответить на научном языке, но я буду вас репортить и играть в уебищные подростковые подкольчики пикчами - агагага как смешно, я вас подколол, агагага


малыш, ты тредом ошибся, пиздуй отсюда, я тебе уже третий раз повторяю. есть что по теме - ты нам пиши, а пока разговор окончен.
118 464996
>>64995

>КУДАХ БАБАХ КУКАРЕКУ


Этот школодебил совсем порвался, замените.

>нам


Оно еще и шизик, хаха.
119 465011
По-хардкору поясните за исследования методов поиска внеземных цивилизаций на данный момент.
120 465013
>>65011
Миша, ты?
121 465015
Были ли в истории науки эксперименты по аберрации света от взаимонеподвижных источника и приемника находящихся на поверхности Земли? Накидайте ссылок на эксперименты если есть.
122 465017
>>65015
Миша, ты?
123 465028
>>65017
Маша, ты?
124 465031
>>65011
На снимках Марса не нашли макдональдсов и нефтянык вышек, а на Землю никто не прилетел с фанфарами и пасом => люди одни во вселенной.
Разве можно спорить с такой логиков?
125 465036
>>65031
Фурфаг, ты?
126 465037
>>65015
Какая может быть аберрация если приемник неподвижен относительно источника, за счет чего она возникает?

Ну был такой эксперимент Майкельсона, например, он показал, что движение Земли никак не влияет на движение лучей света в системе отсчета, связанной с Землей — вне зависимости от положения интерферометра относительно направления движения Земли фотоны по нему летели одинаково.
127 465038
>>65011

>По-хардкору поясните за исследования методов поиска внеземных цивилизаций на данный момент.


Есть маняуравнение Дрейка, где половина аргументов взяты с потолка.
Есть уверенность, что инопланетяне пытаются связаться с нами непременно наиболее привычным нам способом: взять огромную антенну и ёбнуть в сторону потенциально обитаемой звезды Hello World на УКВ.

Кароч, сидят папуасы на острове и пускают дымы в небо, потому что по слухам внук вождя нахуячившись айяхуаски однажды увидел в небе железную птицу, а шаман придумал, что небесные люди живут на облаках и, если им подать сигнал, привезут нямку.
128 465039
>>65036
Нет. При чём тут фуррфагство вообще? Это /спц/, а не /фур/
sage 129 465040
>>65039
И правда не ты а жаль
130 465041
>>65037

>Какая может быть аберрация если приемник неподвижен относительно источника, за счет чего она возникает?


Если фотон движется как волна в среде, то он может сноситься этой средой, если она движется относительно поверхности Земли.

У Майкельсона интерферометр был, а он типа чувствителен на разность фаз, получающуюся от прохождения светом разных путей - это как бы сложнее чем просто померить отклонение луча света. А значит логичнее ловить сначала аберрацию, чем интерференцию. Какие опыты по этой теме проводились и с каким результатом?
131 465042
>>65041

>Если фотон движется как волна в среде, то он может сноситься этой средой, если она движется относительно поверхности Земли.


Да и частица может тоже сносится средой, только меньше.
132 465046
>>65041
Крошечные изменения скорости мерять гораздо проще, чем крошечные отклонения направления, поэтому и использовались интерферометры. В любом случае, никакого сноса не было обнаружено, скорость света во всех направлениях одинакова.

В 21 веке даже троллить эфиром и тупостью в целом — дурной вкус.
133 465047
>>65046
В интерферометре разность фаз может компенсироваться за счет сокращения тел, а аберрация никак не скомпенсирована, вроде.
134 465049
>>65047

>В интерферометре разность фаз может компенсироваться за счет сокращения тел, а аберрация никак не скомпенсирована, вроде.



Каким образом, какие именно тела сокращаются, у тебя шизофазия, что ли?

Сокращение тел пропорционально √(1-v²/c²), а разность фаз в эфирном манямирке должна быть пропорциональна v/((c+v)(c-v)), т.к. сначала свет движется вперед со скоростью c+v за время 1/(c+v), а потом назад со скоростью c-v за время 1/(c-v), складываем и получаем 2v/((c+v)(c-v)).

Тут не надо быть семи пядей во лбу, чтобы понять, что это разные формулы и при повороте интерферометра вокруг своей оси (и соответственному изменению проекции v на направление движения) они будут изменяться по разному закону и тут же перестанут компенсировать друг друга.

Еще один такой пост и я не буду тебя кормить до следующего переката, придумывай что-то получше.
изображение.png1,6 Мб, 1280x720
135 465061
Слушайте.
А у нас, у людей, хоть одна фотка звезды какой-нибудь кроме солнца есть?
Не просто точка, а хоть что-нибудь.
Или не существует телескопов с таким зумом, чтоб хоть что-нибудь сфоткать?
136 465063
Действительно были такие наработки? Или это просто чьи-то фантазии?
137 465066
>>65063
В самом деле НАСАвский концепт, но не более.
138 465067
>>65063
Что-то среднее между твоими вариантами, просто концепт.
http://spaceflighthistory.blogspot.com/2017/02/nasa-johnson-space-centers-shuttle-ii.html
139 465068
>>65049

>этот залетный деревенский шизик из обо/сцая - >>65047


>у тебя шизофазия, что ли?


Похоже ты начинаешь о чем-то догадываться, хаха.
140 465069
>>65046

>В любом случае, никакого сноса не было обнаружено, скорость света во всех направлениях одинакова.


это потому что замеры производились в плотной среде, то есть в нижних слоях атмосферы. тут воздух препятствует свободному движению плотноприжатых сфер. замеры нужно производить в вакууме, на значительном удалении от поверхности земли.
141 465072
>>65046

>Проводить эксперименты даже на ГСО


>Что-то утверждать



Вспомнились сенсации про странное ускорение вояджеров.
142 465073
>>65069
Уроки-то хоть сделал?
143 465080
>>65049

>разность фаз в эфирном манямирке должна быть пропорциональна v/((c+v)(c-v))


Ты тут, как минимум, с размерностями напутал. И теории в добавок не знаешь.
144 465082
>>65049
Лоренц как раз объяснял неудачу Майкельсона реальным сокращением длин в направлении движения ветра.
145 465083
я знаю ещё одну фишку, каждая солнечная система состоит из вихря плотноприжатых сфер и на границе этих вихрей свет преломляется. то есть мы получаем на небосводе обыкновенный калейдоскоп, в котором каждая звезда или галактика переотражается сотни раз, под разными углами и с разной светимостью.
то есть в реальности большая часть звезд это фейк, мираж, их нет на самом деле. и тем более прикольно наблюдать как астрономы дрочат на эти миражи, изучают их. хотя какая-то часть реально существует, но это малая доля.
146 465084
>>65082

>в направлении движения ветра


ебать, да нет никакого ветра, эфир у поверхности земли существует в виде замкнутых колечек, у него скорости валом, а молекулы мешают движению.
чтобы смещение измерить нужно в космосе замеры делать, но даунам это не понять. это же секта отрицателей эфира, которые дрочат на ебштейна и веруют в мифические поля, что сродни полтергейстам или духам, ведьмам.
147 465088
>>65083
>>65084
Ацюковский - типичный пример старческой деменции, маразм в медицинском смысле.
Ну а ты такой с рождения.
148 465089
>>65084

>нет никакого ветра у поверхности


Миллер, Маринов и другие товарищи с тобой не согласились бы.

>нужно в космосе замеры делать


Да, было бы не плохо, там влияния атмосферы нету, скорость ветра должна быть не хилая, по идее.
149 465090
>>65088

>Ну а ты такой с рождения.


гений, великий реформатор и страх всех ебштейновских сект, да?
ну а ты у нас кто? учебникодрочер? что малышу дали в ротик, то он и сосет? типичный быдлопотребитель без пытливого ума и страсти к познаниям - ясно)
150 465091
Надеюсь, мочератор скоро очнется и снесет весь тред, кроме оп-поста.
151 465112
достаточно снести оффтоп школодрона:

>Уроки-то хоть сделал?


>Этот школодебил совсем порвался


>Миша, ты?


ну и в том же духе...
152 465119
>>65090

>учебнико


>сосет


>быдлопотребитель


Школьник as is, типичнейший и обычнейший.
>>65091

>кроме оп-поста.


Wrong. Нахуй этот тред вообще, отстойник для дебилов в спейсаче не нужен уже и так есть треды машкодроча и покпосмос
153 465120
>>65112
Оно обиделась, хаха.
1551297136521[1].jpg121 Кб, 1566x1196
154 465156
Объясните, что такого в этом твитте, что его на 4чане репостят?
155 465157
Это, космическая радиация.
1)Нахуй она нужна?
2) На скольно она мешает бороздить космос?
156 465162
>>65157

>Нахуй она нужна?


Хороший вопрос.
Задай его себе, девочка.
157 465163
>>65162
Но я ппишёл в тред тупых вопрососв, отвечай.
158 465166
>>64812
Гугли "тонкая настройка вселенной". Это не ответ на твой вопрос, но дискутивное направление научной мысли +/- параллельное твоему замешательству. Все это ебола, конечно, скорее из области научной философии, нежели научного подхода, но раз тебе интересно...
159 465167
>>64821
Нихуя ты умный. Ну давай, расскажи мне за фундаментальную разницу между гравитационной постоянной и отношением диаметра окружности к ее длине, особенно с учетом того, что число пи применяется в физических формулах направо и налево, подразумевая собой ровно такую же, как и те, что ты называешь физическими константами, характеристику мироустройства.
160 465172
>>65163

>я ппишёл


Ути какая няка :3
161 465173
>>65167

>отношением диаметра окружности


Эйлеровскую приколюху в твоей шкалке еще не проходили значит.
Ясно.
Понятно.
162 465179
>>65167
Отношение диаметра окружности к длине в евклидовом пространстве справедливо абсолютно в любой Вселенной. Оно никак не зависит от параметров физического мира, а только от аксиом геометрии. Кроме того, это отношение для реальной окружности отличается от пи, хотя и на какие-то ебически неуловимые доли, потому что Вселенная искривлена и нихуя не евклидова. Оно применяется в физических формулах, вот только в физике налево и направо пренебрегают мелкими погрешностями.

А вот гравитационная постоянная ниоткуда не следует. Ну или, по крайней мере, мы не знаем, откуда.
163 465182
>>65179

>от аксиом геометрии


Еще один.
164 465203
>>65119
Если бы ТТВ не было, половина этих самых тупых вопросов появилась бы отдельными тредами.
165 465210
>>65203

>половина этих самых тупых вопросов появилась бы отдельными тредами.


Банить к хуям, как минимум на месяц.
Быдлодебилы не нужны вообще, очевидно на примере этого треда.
166 465211
>>65210
Вахтёр, плиз. Если и банить, то таких, как ты
15484156567600.png474 Кб, 748x636
167 465213
>>64788

>Меркурий

168 465226
Почему все кратеры круглые? Они такими могут быть быть только при прямом вертикальном падении. А они все всегда падают под углом, значит, и кратеры должны быть яйцеобразной формы с ударной воронкой ближе к узкой части.
169 465227
>>65226

>Они такими могут быть быть только при прямом вертикальном падении


Дебил, блять.
170 465230
>>65211
Ну можно еще ниграми завайпать.
Лично тебе что ближе?
171 465231
>>65226
Потому что скорость удара настолько высока что происходит взрыв
172 465232
>>65231
Пиздец ты гнида.
174 465236
Почему звёзды и планеты круглые если должны быть дискообразные?
himki.webm342 Кб, webm,
480x360, 0:05
175 465237
>>65236
пацаны, это представитель общества плоской земли! не отвечайте дауну!
глаз-урагана.jpg190 Кб, 502x314
176 465239
>>65236

>Почему звёзды и планеты круглые если должны быть дискообразные?



потому что солнечная система представляет из себя вихрь из плотноприжатых сферообразных частиц эфира. поскольку, как я писал выше - пустота невозможна, то все промежутки между сферами заполняет идеальная жидкость. сферы отбрасываются наружу, а излишки идеальной жидкости собираются в центре вихря в виде крупной капли, которую мы называем солнце.
звезды и наше солнце шарообразные, потому что шар это самая энергетически выгодная форма.
но это не всё. вихрь из сферообразных частиц работает как гигантский насос и отбрасывает идеальную жидкость наружу, откуда она благополучно засасывается на полюсах других солнечных систем. и в наше солнце на полюсах засасывается идеальная жидкость. там она приобретает единую скорость и поэтому объединяется в общий столб, а поскольку она проходит через ось, образованную сферообразными частицами, то столб приобретает грани и закручивается по оси. так рождаются нейтрино и прочие винтообразные частицы со спином 1/2. сфера может оставаться на месте, а винтообразная частица всегда должна лететь вперед.
когда винтообразные частицы входят в солнце, то они отламываются на короткие частички, сталкиваются в центре солнца и разлетаются к экватору. крупные бракованные частицы не могут пройти в промежутки между сферами и образуют темные пятна. Если вихрь крупный, то темные пятна разрушаются, а если мелкий, то копятся на поверхности пока полностью не застилают звезду. так образуются планеты. поэтому мы не можем увидеть маленькие звёзды, они все спрятаны. признаком, что внутри планеты находится звезда, является наличие общепланетарного магнитного поля. поскольку магнитное поле порождается винтообразными частицами, которые вылетают из звезды. поэтому планеты тоже круглые. чем меньше звезда, тем более тяжелые атомы образуются при дроблении темных пятен.
177 465240
>>65239
иди нахуй
178 465241
>>65240
репорт
179 465242
>>65239
>>65240
>>65241
Два дебила - это сила.

И они еще говорят что ТТВ нужен.
180 465253
>>65237
Просто по логике центробежная сила должна размазывать тела в блинчик, а этого не происходит, чому?
image.png132 Кб, 600x499
181 465254
>>65253
А балерину почему центробежная сила не размазывает в блинчик?
182 465266
>>65254

>А балерину почему центробежная сила не размазывает в блинчик?


Ну это смотря до каких скоростей её раскрутить)
183 465271
>>65254
А ничего что юбка стала блинчиком?
184 465275
>>65271

>А ничего что юбка стала блинчиком?


это не юбка - это гироскоп.
185 465281
Как вы относитесь к строительству офиса Роскосмоса за 25 миллиардов рублей?
186 465283
>>65281
В соответствующем треде кормить перестали?
187 465285
>>65275
это пояс астероидов как у сатурна
188 465286
>>65266
воооот, твоя извилина начинает что-то подозревать
189 465337
Что с этим тредом не так? Почему у меня ощущение, что я в /б, даже привкус характерный появился.
190 465338
>>65281
Космическим масштабам и распил соответствующего уровня.
191 465364
Почему нельзя направить Хаббл на Луну и сфотографировать следы астронавтов?
192 465373
>>65364
Разрешения не хватит, Луна слишком далеко. Даже следы на Земле еле-еле хватило бы разрешения разглядеть, но атмосфера мешает.
193 465374
Есть ли скорость, быстрее скорости света?
194 465402
>>65337
>>65338
>>65364
>>65373
>>65374
Почему не в школе?
195 465411
>>65402
Читать умеешь? "Тред тупых вопросов". В ОП-посте не показан предел тупости вопросов. Так что давай-ка ты или держи свой обоссаный рот закрытым или отвечай по делу на вопросы.
196 465415
>>65402

>спросил ударник Дмитрий, скролля двачик на телефоне под партой.

197 465420
>>64509
Есть ответ на этот вопрос?
198 465436
Я тут подумал, ведь можно же создать цепь замкнутого цикла и жить на других планетах. Можно было бы даже подземные города строить.
199 465438
Если у Бурана движков не было, зачем его сбоку цепляли, как Шаттл?
200 465440
>>65438
Украли компоновку не задумываясь.
201 465470
>>65420

>While gravitationally lensed light sources are often shaped into an Einstein ring, due to the elongated shape of the lensing galaxy and the quasar being off-centre, the images form a peculiar cross-shape instead.[3]


Короче, обычные еврейские отговорки.
202 465472
>>65436
Построй сначала город на дне океана, глубоко под землей, в воздухе и на полюсах, а потом суйся осваивать другие плонеты.
203 465473
>>65436

>создать цепь замкнутого цикла и жить на других планетах


Мы на земляхе пока такое не можем провернуть, что уж говорить про другие планеты. На, почитай: https://ru.wikipedia.org/wiki/Биосфера-2

Особенно вот это:

>Однако через несколько недель жизнь людей, живущих натуральным хозяйством, нарушилась. Микроорганизмы и насекомые стали размножаться в неожиданно больших количествах, вызывая непредвиденное потребление кислорода и уничтожение сельскохозяйственных культур (использование ядохимикатов не предусматривалось). Обитатели проекта стали терять в весе и задыхаться. Учёным пришлось пойти на нарушение условий эксперимента и начать поставку внутрь кислорода (23 тонны) и продуктов (эти факты скрывались и были разоблачены впоследствии). Первый эксперимент закончился неудачей: люди сильно потеряли в весе, количество кислорода снизилось до 15 % (нормальное содержание в атмосфере — 21 %).

sage 204 465486
>>65470
Иди нахуй.
>>65472
Иди нахуй.
205 465492
>>65374
Есть. Скорость сближения двух фотонов, например, летящих навстречу друг к другу, равна как минимум 2c - это очень легко экспериментально проверить.
206 465501
>>65492
Ньютон, плиз
207 465552
Вопрос, если метан и кислород в жидком состоянии, то можно ли просто открыть вентиля и подать на камеру сгорания?

Будет ли выше кпд если камера сгорания будет обогревать жидкий метан и кислород?

И какая будет разница в кпд по цирфам если сравнивать с подачей на 150 атмосферах? 40% разницы или 400%?
208 465561
Если б космос проводил звук, какой бы он был ну кроме ты хуй/мамку ебал, само собой? Насколько громкий, например?
209 465580
>>65561
Твой вопрос взаимоисключающий. Звук зависит от среды, в которой он распространяется. В космосе звук не распространяется, потому что такой среды нет.
Это всё равно что спросить, как плавается в осушенном бассейне. Никак блядь.
210 465605
>>65580
Ну, во-первых, среда там есть и звук тоже. Но меня всё-таки интересует обычный воздух, но чтоб не гравитировал никуда.
https://www.youtube.com/watch?v=f_a9QqzCF14
211 465667
212 465687
>>65552
Слесаря отроют вентиля, вспыхнут огня и наши корабля выйдут на орбита.
1+.png33 Кб, 845x732
213 465692
Поясни за мою идею.
Когда мы в космосе проблема утечки состоит в том, что при нарушении герметизации из-за перепада температур воздух утекает.
В сайфае-космоопере "Звёздные Войны" эта проблема решается некими полями, позволяющими проникать кораблям сквозь этот барьер, но не дающая утечь воздуху.
И я подумал, а что если дать утечь воздуху? Ведь воздух будет капельками прилепать к стенкам корабля, особенно если это узкие стенки в туннеле шахты. В космосе и воздух будет иметь определённый уровень поверхностного натяжения (как мне кажется). То есть, имея трубу определённой длины - весьма большой, как я полагаю - можно создать один небольшой открытый шлюз, позволяющий держать открытым гипотетическую станцию для прямого доступа в открытый космос, не боясь при этом растерять драгоценный воздух и не используясь при этом сайфай допущениями волшебный силовых полей, а ограничиться реальной физикой движения потоковых масс и частиц. Что скажешь, анон? Подозреваю, идея моя легко допустима, если будет туннель длинной в десяток другой километров, заворачиваемый так и сяк, и имеющий какую-то определённую поверхность внутренней стороны, соблаговаляющую удержанию воздуха.
214 465693
>>65692

>туннель длинной в десяток другой километров


Ну и какой смысл в такой херне, если можно просто шлюзы поставить на всём протяжении и укоротить в 100 или 1000 раз? Алсо, воздух будет утекать из этой штуки просто потому что давление будет стремиться выровняться между всеми условно говоря "сосудами", может это будет происходить медленно, не быстро, но это и не должно происходить быстро, просто банально из-за длинны этой ебаторики на несколько километров и объемов газа в ней, я боюсь представить какого размера этот шарик с 1атм в нижней части картинки.
1+.png23 Кб, 845x732
215 465697
>>65693
А если так и за счёт вращения отталкивать воздух?
Ну и хуй с ним - щепотку силовых полей как в ЗВ (только я х/з, что это за поля должны быть)

> Ну и какой смысл в такой херне


Чтобы при каждом открытии-закрытии шлюза драгоценный воздух не выбрасывало под силой собственного давления. При многократном использовании даже утечка пары млЛ воздуха в долгосрочной перспективе приведёт к иссяканию воздуха внутри до непригодных для жизни величин

> я боюсь представить какого размера этот шарик с 1атм в нижней части картинки.


Форма сперматозоида - условность. Если проще представить, возьмём некую маленькую Звезду Смерти (не по функционалу, а по размерам) и сделаем так, чтобы один туннель наматывал круги по поверхности превращая всю поверхность в огромный лаз со входом из космоса например на внешней стороне северного полюса, и безшлюзным выходом во внутреннее убранство станции на внутренней стороне южного полюса.
216 465712
>>65561
Всё бы шумело и пердело отовсюду. Представь, какой хотя бы от звёзд шум бы издавался. Вряд ли в таких условиях у организмов бы вообще слух развился. Я думаю, было бы что-то вроде того же белого шума
217 465724
>>65697
Создашь гравитацию за счет вращения и выйдет то же самое, что и у нас на Земле — даже с 1g атмосфера тянется на сотни километров вверх и все равно по чуть-чуть утекает наружу в космос.

Чтобы утечку снизить до уровня погрешности, тоннель должен быть сотни километров длиной, при том, что можно построить пару шлюзов длиной в несколько метров и добиться того же самого эффекта.

>Чтобы при каждом открытии-закрытии шлюза драгоценный воздух не выбрасывало под силой собственного давления



Какого давления? У нормальных людей шлюз герметично изолирован как от космоса, так и от основного объема станции, и после закрытия дверей воздух из него откачивают.
218 465744
>>64488 (OP)
Солнце погасло нахуй.
Ну а если серьезно, там солнечная активность существенно просела. Что это значит для Земли? Какие могут быть последствия? Как часто такое происходит? Когда ожидается восстановление активности?
219 465746
>>65744
Ничего, никакие, каждые 11 лет, в следующем году.

https://www.spaceweatherlive.com/en/solar-activity/solar-cycle
https://en.wikipedia.org/wiki/Solar_cycle
220 465818
Будут ли запускать для луны/марса аналог жпс при колонизации? Есть ли уже какие-то проекты?
221 465821
>>65061
Бамп?
223 465857
>>65853
Спс. Я так понимаю, там нужно кучу человек поселить, чтобы развертывание было оправданным.
224 465869
>>65724

> и после закрытия дверей воздух из него откачивают.


Как? Там же уже вакуум. Не будешь же ты мне втирать про вакуумные вентиляторы, которые дуют эфиром на водяную пыль.
225 465922
>>65821
>>65061
Есть, аж пара десятков.
https://en.wikipedia.org/wiki/List_of_stars_with_resolved_images

Но помимо общей формы нихуя особо не разглядеть.
226 465930
>>65869
Где вакуум? Когда шлюз закрывают с внутренней стороны, в нем одна атмосфера, как и в самой станции, которую потом откачивают обычными вакуумными насосами.

До <0,01 Па несколько кубометров можно откачать за вполне разумное время.
228 465949
>>65930
Нет, подожди.
Значит, капли воздуха дрыгаются в вакууме космоса. Есть вакуумный насос. Какие образом частицы будут в вакуумный насос идти по тяге, если нет разницы давления между вакуумом пылесоса и вакуумом космоса?
229 465973
>>65818
Конечно будут. По сравнению со стоимостью колонизации (которая еще хуй знает когда будет), накидать пару десятков автономных спутников - считай что бесплатно.
15513614264520.png1,5 Мб, 1430x645
230 466058
Что на пике?
231 466063
232 466065
>>66058
Макет спускаемого аппарата Венеры-4.
233 466066
>>66065

>Венеры-4


Спасибо...
234 466067
>>66058
А есть ли побольше фоточек этой штуки с разных ракурсов?
235 466171
>>65492

>Einstein edition


>этот пост

236 466174
>>65746

>Ничего, никакие, каждые 11 лет, в следующем году.


а солнечная активность имеет периоды в 11 лет потому что идеальная жидкость засасывается на полюсах солнца и выбрасывается по экватору через промежутки между плотноприжатыми сферы эфира. но выбрасывается уже в виде спиралевидных частиц с гранями, бракованные частицы не проходят в промежутки между сферами и остаются на поверхности солнца, формируя темные пятна.
когда поток идеальной жидкости в полюса большой, то и пятен много, а когда поток слабый, то пятен мало.
237 466185
Я не нацсрача (да и вообще другого любого срача) ради, а просто так аккуратненько, корректненько так, мягонько и ненавязчиво так, никого ни в чем не возвышая и не принижая так спрашиваю - был ли полет Бурана чем-то охуенным и выдающимся с технической точки зрения?
Мог ли Спейс Шаттл садиться на автопилоте?
238 466188
>>66185
Хуета полная. Спиженный с Шатла планер и почти вся начинка. Что тут выдающегося? Автопосадка? Охуеть новинка. К моменту полета Бурана самолеты уже лет 30 умели сами садиться.
Буран - это тупое говно тупого говна. Почему думаешь на него забили после единственного полета? Потому что это тупое говно тупого говна.
239 466211
>>66185

>был ли полет Бурана чем-то охуенным и выдающимся с технической точки зрения?


Нет, не был.

>Мог ли Спейс Шаттл садиться на автопилоте?


После небольших доработок мог, но сделано этих доработок не было, в основном потому, что попросту не было запроса от НАСА.

Если кто-то говорит, что не мог, то отвечай, что ну тогда Буран не мог ни выводить грузы, ни возить людей, т.к. для этого требовались доработки гораздо большего масштаба.
240 466214
>>66211

>Буран не мог ни выводить грузы


Но он вывел.
241 466215
>>66067
Помогите пожалуйста.
sage 242 466217
>>66215
Google -> Венера-4 Мемориальный музей космонавтики
243 466220
>>66185
В шаттле по идеологическим причинам было сделано так, чтобы некоторые решения нельзя было принять без человека. Например, выпуск шасси. Так-то он обычно всё равно садился на автоматике. Так что да, шаттл мог сесть автономно, но только на брюхо. Это такие свои загоны были у руководства наса в то время.
244 466227
>>66220
А в чем смысл, чтобы совки каким-то образом не спиздили управление чтобы посадить шатол к себе?
245 466233
>>66227
Чтобы у астронавтов не было ощущения, что за них партия решает, когда выпускать шасси.
246 466249
>>66227
На ручном управлении были только четыре так называемых "необратимых" действия, чтобы астронавты не бугуртили: выпуск шасси (которые не имели приводов и тупо выпадали через щит, обратно не задвигались), выдвижение приемника воздушного давления (обратно не задвигался), выпуск тормозного парашюта (аналогично) и включение вспомогательных силовых установок.

Срабатывание любого из них раньше времени (кроме разве что ВСУ, где можно было очухаться до того, как они выработают весь гидразин) фактически гарантировало при посадке катастрофу а-ля Колумбия, поэтому сами астронавты были против их автоматизации.
247 466254
>>66214
Ну и что он вывел?
248 466257
>>66254
Модуль 37КБ.
249 466292
>>66211

>После небольших доработок мог, но сделано этих доработок не было


Было. После катастрофы "Колумбии".

Добавили специальный кабель, что-то там с чем-то соединявший, чтобы с земли можно было выдать команды, перечисленные в >>66249

Сделали чтобы в случае чего оставить астронавтов куковать на МКС, а корабль попытаться посадить на автомате.
250 466293
А что это за трэд?
251 466342
Если б возле сталкивающихся чёрных дур крутилась насквозь промороженная планета, то гравитационные волны мгновенно бы её равномерно прожарили, а то и испарили? Или они слабо передают энергию?
252 466386
Тёмная энергия действительно имеет массу со всеми атрибутами (инертность/собственное притяжение/всякие ото-эффекты) или это просто не имеющий (пока что?) физсмысла эквивалент из е=мц2?
253 466387
Ну и тогда уж, может ли тёмная материя, например, не иметь инерции, а только притягивать? Рассматриваются такие варианты?
>>66386
254 466399
>>65236
Потому что гравитация.
255 466400
>>66399
Чет я подумал, а почему всё вращается?
256 466447
Можно ли раскачать станцию изнутри, чтобы корабль не смог пристыковаться?
257 466482
>>66447
Дмитрий Олегович, вы законы сохранения импульса и момента импульса в школе проходили?
258 466642
>>65226
Неприятный вопрос, потому что это таки взрывы и не факт, что от метеоров.

Как пример против метеоров падение оного под Хабаровском.
259 466677
>>65231
Это по сути застывшая волна. Энергии выделяется столько, что твёрдое вещество начинает вести себя как жидкость.
260 466680
Почему у НАСА не было автоматической стыковки (до событий последних часов), хотя Совок уже лет 50 умеет в автоматическую стыковку? Казалось бы, нация с самыми мощными компьютерами мира, а в автоматическую стыковку не могут.
Тоже боятся за "слишком самостоятельные" технологии, как описано выше про Шаттл?
262 466730
>>66693
А не словесного, а эмпирического доказательства не будет что ли? Смысл в этих разглагольствованиях?
263 466731
>>66730
Там две трети ролика рассказывается об экспериментах Роскосмоса НАСА на эту тему, как физических, так и виртуальных, на симуляторе.
264 466834
>>64488 (OP)
Почему нигде нет видео земли с орбиты?
Типа как в гугл картах снимки с космоса, только видосы. Чтоб смотреть как люди ходят, самолеты летают, etc.
265 466884
>>66834
С МКС уже дохуя лет стримят видео, смотри не хочу.

В большом увеличении снимать смысла нет, спутники слишком быстро движутся и точка съемки моментально сместится, никакой полезной информации из такой съемки не извлечь и это нахуй никому не надо.

>Чтоб смотреть как люди ходят


Максимальный уровень приближения на гугл картах, где хорошо видно людей, это вообще обычно аэрофотосъемка, а не фото со спутника, со спутника качество подерьмовее.
266 466892
>>66834

>Почему нигде нет видео земли с орбиты?


есть, плохо искал. Есть целые коммерческие компании со своими собственными спутниками, которые снимают видео любой точки Земли на заказ.
267 466895
>>66834
Ты слишком веришь голливудским фильмам, где со спутников не только видео снимают, но и разговоры подслушивают. На деле спутник хуярит по орбите со скоростью ~7 километров в секунду, город уровня Москвы он пролетит буквально за пару секунд и разумеется что сфокусироваться можно только на какой-то конкретной области и только для съемки фотографий. Тут еще надо заметить, что спутник летает на полярной орбите, а Земля под ним вращается, то есть пролетев один раз над Москвой, в следующий раз он в этой области может оказаться и через неделю и через месяц, как повезет.
268 466997
>>66895
А если геостационарный спутник? Или слишком далеко?
269 467003
>>66997
Ну ёпта, конечно далеко. 35,786 километров, а спутники ДЗЗ летают в районе 500 где-то, плюс-минус пара сотен км в зависимости от конкретных целей.

На геостационаре спутники ДЗЗ тоже есть, но специфические. Некоторые погодные, например. https://en.wikipedia.org/wiki/Weather_satellite#Geostationary
270 467120
>>66680
С одной стороны, ручную стыковку, с содействием со стороны компьютера, проще сделать.
С другой стороны, в штатах никогда не испытывали каких-то особых проблем с использованием пилотов. Это скорее наоборот, у СССР было маниакальное стремление сделать пилота ненужным.
271 467136
Если гигантский космический великан проткнет земную мантию гигантской соломкой и выпьет всю мантию, будет ли земная кора, литосферные плиты держаться друг на друге как кирпичи в арке, или потрескается и упадет на ядро?
272 467161
Первая ступень Флакона + Дракон на ней, можно ли такую связку использовать как мини-BFR для межконтинентальных перелётов?
273 467162
>>67136
Тебя это ебать не должно
274 467163
>>67003

>Ну ёпта, конечно далеко


Просто ты не в курсе возможностей ЦРУ.
275 467166
>>67163
Ну да, у ЦРУ законы физики свои, секретные.
sage 276 467168
>>66731
Хуле ты ему отвечаешь? Эта зелень пытается скатить тред ТВ на уровень зогача.
277 467169
>>67168

>поднять тред ТВ на уровень зогача.


Поправил, не благодари.
278 467171
>>67169
Уриноприемник, уймись.
279 467172
>>67171
Ты докукарекаешься до вайпа раздела и утопления символа доски - ЖУКОВ-ГОВНОВОЗОВ
280 467175
>>67136
Рухнут даже без всяких трещин, такую массу хуй удержишь. Да хоть ты кору замени на графен - все равно рухнет.
281 467177
Вопросы:
1. Правда что на МКС пьют переработанную мочу? Т.е. замкнутый цикл воды.
2. Как моются на МКС?
282 467181
>>67177
1. Пидора вопрос.
2. Да.
283 467194
>>67136
Ну эта хуйня будет гравитационно нестабильна в любом случае.
sage 284 467201
>>67172
Иди нахуй.
285 467218
>>67172
Навозников нетрож!
286 467232
>>67218
На коленях умоляй, как положено.
И за этого шароеба тоже - >>67201
287 467238
>>67161
Только если в одноразовом режиме, с реюзом на межконтинентальный перелет + торможение и посадку первой ступени не хватит дельты. В самом драконе топлива маловато, он слабый помощник.
288 467240
>>67161
Такие перелеты это 90% орбиты, не хватит.
289 467252
>>67177
1) Только американцы. Русские же резонно считают что таким образом можно зашкварится а и пьют ювенильную воду из пыли, которая как роса оседает на солнечных панелях.
2) Обливаются потом и это смывает грязь. Кстати американцы потом и это пьют.
290 467254
>>67177
1 такая вода используется только в технических целях и вообще это скорее отработка технологий
2 салфетками обтираются
291 467262
Да что уж там, мы даже тут на земле пьем то, что когда-то было чьей-то мочой.
292 467263
>>67252
>>67254
>>67262
Мочехлебы с мочепараши о наболевшем.
largefairingcomparisongraybg.jpgthumb.jpg13 Кб, 264x400
293 467612
Почему редко используют такие большие обтекатели и странные формы?
294 467615
>>67612
А - аэродинамика
323231.png114 Кб, 680x521
295 467618
А можно ли сделать ракету на метане и кислороде под 200 атмосферами в баллонах из композитов?

КПД будет не очень, но с другой стороны можно без нагнетателя обойтись(тоже вес)
СпецГО Космос-3М.jpg36 Кб, 800x600
296 467630
>>67612
Обтекатели, отличные от круглых в сечении, скорее вынужденная мера, когда есть базовый ГО, но полезная нагрузка в него, сука, не влезает.
297 467693
Тёмная энергия является причиной энтропии вселенной?
298 467695
>>67693
Нет Возможно ты неправильно понимаешь значение слова энтропия
299 467714
>>67695
Вселенная расширяется из-за тёмной энергии -> остывает -> тепловая (энтропийная) смерть. Или что то там с симметриями связано, законами сохранения?
300 467717
Как пар раскручивает турбины?
301 467725
>>67717
Давлением.
302 467739
>>67714
Полна энтропия тепловая смэрт
303 467741
>>67739
>>67714

это лишь равномерное распределение тепла во вселенной, ну и темная материя как и расширение вселенной на это ни как не влияет, только чуть-чуть замедляет случайно отправил
304 467745
>>67741
Почём нынче паскод? Нужно авторизоваться, или кукисы просто не удалять? Сколько раз было "перейдайте Абу произошла грубейшая случаная ошибка"?
305 467746
>>67714
Даже если бы Вселенная была постоянного размера, энтропия бы все равно росла. В некотором роде, энтропия - показатель "хаотичности". Собрал энергию в одном месте - уменьшил хаотичность (в этом месте). Но для этого надо ее увеличить, причем более существенно, в других местах.
306 467747
>>67746
Что является причиной энтропии? В научных термина если не трудно.
307 467756
>>67747
Да.
308 467759
>>67747
Выравнивание температуры
309 467760
>>67747
Ну тащемта это постулат, он ниоткуда не следует. С другой стороны, он хорошо проверен эмирически.
sage 310 467897
>>67756
>>67759
>>67760
Хуйню напейсали.
311 467915
>>67760
>>67759
Какой-то всемирный уравнитель? Какой у него принцип? Плюс-минус-пополам? Принцип наименьшего действия/движения? Что-то такое было но не могу вспомнить. Неужели это просто упругое распределение. Хмм. Кто-бы носом тыкнул.
312 467931
>>67897
Ну напиши нехуйню, сагователь говёный.
sage 313 467934
>>67931
А зачем? Все равно здесь никто ничего не поймет. Про законы термодинамики и их статистическое обоснование здесь, похоже, никто и не слышал. Попробуй прочитать для начала вот это:
https://studfiles.net/preview/882529/page:35/
314 468073
Котаны, вот тепловая смерть. Все развалилось до действительно элементарных частиц (какие бы они не были), максимум энтропии, полная однородность/инвариантность etc.
Но ведь даже в таком состоянии будут присутствовать флуктуации, которые снова приведут к созданию составных частиц, атомов, etc. Короче, кто победит - концепция тепловой смерти или концепци больцмановского мозга?
ZFpNokDwwHQ.jpg42 Кб, 604x576
315 468116
Спейсач, поясни за черные дыры - вот никакой объект обладающий какой-никакой массой, после приближения к чОрной дыре не способен от неё съебаться.
А что с радиоволнами? Они же не обладают массой.
Что если запустить в черную дыру йоба-зонд с кучей разных датчиков, и все это дело транслировать наблюдателю через йоба антенну, то сможет ли наблюдатель получать данные в онлине т.е. за время прохождения радиоволн от зонда до наблюдателя, а не через бесконечно долгое время?
316 468138
>>68073

>которые снова приведут к созданию составных частиц


Эт схуяли?
317 468140
>>68116

>после приближения к чОрной дыре не способен от неё съебаться


После погружения под горизонт событий ващет.

>Они же не обладают массой


Энергия и масса эквивалентны в таких категориях. Само понятие массы слишком комплексное, в которое лучше палкой не тыкать, дабы кефирошизика и прочих адептов Ацюковского не возбуждать.
Судя по уровню твоих вопросов, тебе для базиса не помешало бы по тематическим статьям в википедии хотя бы пробежаться.
318 468141
>>68138
Чисто статистически. Рано или поздно это случится.
319 468309
>>67747
В детстве писал программу, рисующую узор, по алгоритму рядом искался цвет, что на единицу больше и текущий ему приравнивался, для последнего первый считался следующим, то есть цикл замыкался. Пиксели изначально получали случайные цвета, что любопытно, в конченом итоге получались плоские вихри во всём этом деле.

Позже, пытаясь смоделировать так сказать вселенную, что стремится к среднему значению, подобного не наблюдалось, всё быстро схлопывалось в один цвет.

К чему это я, к тому, что энтропии нет, это ересь, есть безконечное перетекание энергий одна в другую.
320 468534
>>68141
Нет, схуяли этот процесс станет доминантным вдруг? Ну образовался больцмановский мозг. Ну обратился обратно в стандартный кисель под действием внешних условий за пару часов-дней. С хера ли вдруг по-твоему флуктуации начнут носить упорядоченный характер, превалирующий над энтропией?
321 468673
>>68309

>безконечное


Уроки-то сделал?
322 468677
>>64488 (OP)
Сколько лет осталось существовать жизни на Земле? Видел оценки от 200 млн, до 1 млрд.
323 468682
>>68677

>Сколько лет осталось существовать жизни на Земле?


Да хуй знает, какие-нибудь бактерии-экстремофилы будут существовать практически до начала превращения Солнца в красный гигант. Разве что полное испарение всей воды может их добить раньше, но думаю под землей, да в районе полюсов останутся какие "влажные" места.

>Видел оценки от 200 млн, до 1 млрд.


Ну, собственно, из-за неоднозначности того, что считать "существованием жизни", такие и разбежки. Опять же, не стоит недооценивать способность жизни приспосабливаться. 200 млн, как мне кажется, это уж больно пессимистично.
324 468683
>>68682
Ну ок, давай будем говорить о многоклеточных организмах. Вот собственно полное испарение воды и вангуют на период

>от 200 млн, до 1 млрд


а хочется конкретики.
325 468685
>>68683
Полное испарение воды только через миллиард лет.
326 468686
>>68683
Ну я имел в виду не только испарение океанов, а полное исчезновение воды из земной коры, по крайней мере из ее верхней части. Бактерии и архебактерии замечательно живут под землей, причем хуй знает, до какой глубины.

А многоклеточная жизнь наверняка вымрет с океанами. До их полного пересыхания возможно распространение современной фауны "черных курильщиков" - эти парни к горячей воде уже привыкли (да и другие могут адаптироваться). Хотя, правда, для еды им нужны бактерии в больших количествах, так что может от "курильщиков" уйти и не смогут.
изображение.png660 Кб, 1200x675
327 468687
Какой смысл в Ангаре А5\А7 если она выводит в 2 раза меньше чем Falcon Heavy, того же класса. Это ведь заранее проигрышный вариант, вкладывать в продолжение её разработку, если она неспособна конкурировать на рынке и заранее устаревшая...

Объясните
328 468692
>>68677
Откуда ты увидел такие оценки и чем они аргументируются?
329 468694
>>68687
Нихуя они не того же класса. А5 примерно как фалкон, А7 чуть больше, ну как полтора, но никак не три. Но в остальном ты прав, никчемная ракета, хуй знает зачем делают, позорятся только.
330 468697
>>68687
Грузоподъёмности у вариантов Ангары подходящие, не в них дело.
x4df18864.jpg56 Кб, 600x509
331 468761
Фапают ли космонавты на мкс? Что будет, если космонавт обмалафится во сне от длительного воздержания или пернет с подливой? Или они там в подгузниках круглосуточно, что бы последствия таких конфузов по всей станции не летали?
332 468768
>>68761

>Что будет, если космонавт обмалафится во сне от длительного воздержания или пернет с подливой


Его с позором снимут с МКС ближайшим кораблем.
333 468770
>>68761

>Фапают ли космонавты на мкс?


Падалка сказал, что да, фапают, точно так же, как полярники и подводники.
334 468789
>>68770
В рот друг другу?
335 468797
>>68770
А почему им не завезут резиновую куклу или хотя бы мастурбаторы? Это же МКС, технологии, хайтек, а они там письку дергают как в каменном веке.
336 468804
А что будет если набрать первую космическую скорость если лететь не по орбите а тупо вверх от земли?
337 468806
>>68804
Ебнешься обратно через какое-то время. Чтобы улететь кхуям нужна вторая космическая.
338 468807
>>68806

А на какую высоту можно попасть?

Какие кстати высоты для 1,2,3,4,5км в сек если вертикально переть?
339 468808
>>68807
Лень считать, но на большую. Десятки тысяч километров, если так, с потолка.
340 468812
Считается, что в галактиках дохуя темной материи, которая влияет на их структуру и скорости орбитального (вокруг центра галактик) движения звезд. Но для рассчетов всяких орбит внутри Солнечной системы никто никакую темную материю в рассчет не принимает, при этом точность охуенная. Как так? В Солнечной системе нет темной материи? Или она сидит внутрях скоплений обычной материи (Солнце, Юпитер) и поэтому похуй, какая доля массы там на что приходится?
Erdgvarp.png12 Кб, 290x222
341 468813
>>68807
>>68808
Если прикинуть при скорости 1 км/сек, пренебрегая сопротивлением воздуха и считая g = 10 м/с2, получится 50 км. g на самом деле чуть меньше (пикрелейтед), а сопротивление воздуха у Земли на такой скорости просто нахуй всё расплавит. Если взять 8 км/с, считать, что стартуем с высоты 100 км, и взять g = 8 м/с2 (то есть для высоты ~700 км), то будет 4000 км, что, естественно, гораздо меньше реального значения, потому что там уже g будет заметно меньше.
342 468814
>>68812

>В Солнечной системе нет темной материи?


Зис скорее всего по теории. В тёмную материю есть несколько кандидатов, и звезды главной последовательности кандидатами не являются.
Screenshot20190309182426.png42 Кб, 856x627
343 468819
>>68808

>Лень считать


Сейчас прикинул дифур: r'' = -cr^-2, где r - расстояние до центра Земли, а c - коэффициент, зависящий от гравитационной постоянной и массы Земли. Пикрелейтед - то, что мне выдал Вольфрам, лол.
344 468825
>>64488 (OP)
Какого хуя научпоп надрачивает на термоядерный реактор?
Мол термоядерные реакторы достаточно мощные, чтобы завести полноценный старшип на ионной тяге. Но современные реакторы деления тоже довольно мощны, основная проблема в охлаждении контура, как решат её при термояде?
345 468826
>>68797

>А почему им не завезут резиновую куклу или хотя бы мастурбаторы?


Подозреваю, что у американцев все это есть, просто не разглашается из этических соображений. А пидорахам оно и нах ненадо, там кузмичи сидят, которые думают только о том, как бы накатить поскорее
https://www.youtube.com/watch?v=28EMnhKd4p8
346 468827
>>68825

Суть в том, что имеется в виду термоядерныей ракетный двигатель, то есть топливо для реактора является рабочим телом ракетного двигателя и уносит тепло с собой. А вообще сигма-деретринитация во все поля.
347 468829
>>68534
Ну вот моя мысль о том, что энтропия в этом случае не изменится, просто в какой-то области пространства появится более стабильная сейчас конфигурация. Примерно как на первых стадиях абиогенеза в первичном супе увеличивалась концентрация нуклеотидов, соединенных в цепочки - так как одиночные нуклеотиды быстро распадались обратно.
Более глобально - не ебут ли квантовые принципы (нулевые колебания физического вакуума) термодинамику - ведь переход в ложный вакуум не меняет энтропию системы, но приводит к изменению состояния.
У менч сейчас 39.8 если что. В таком состоянии я могу только думать.
348 468831
>>68826

>https://www.youtube.com/watch?v=28EMnhKd4p8


Вот первое, что меня удяивляет, что это вообще обнародовали.
А второе, что лахта нигде не кричит, что это фейк.
Из этого напрашаивается вывод, что сие было додобрено на высшем уровне, как утверждающеее русские скрепы и позывы глубинной русской души.
349 468839
>>68831
Просто пидорах готовят к окончательному сворачиванию космической программы, когда очередной "Союз" наебнется. Чтобы не сильно бугуртили, типа а нах нам это все, лучше путинки накатить.
350 468840
>>68789
Твоей мамке за щеку.
351 468843
>>68812
Плюс еще средняя плотность темной материи по Млечному пути более чем на десять порядков ниже, чем средняя плотность Солнечной системы в пределах орбиты Нептуна.

Если темная материя не внутри массивных тел сидит, а более-менее равномерно распределена по пространству, то заметить вносимое ей ничтожное отклонение на таких масштабах фактически невозможно, надо смотреть хотя бы на масштабе нескольких световых лет.
352 468851
>>64488 (OP)
Почему на втором оппике получается КРЕСТ ЭЙНШТЕЙНА, хотя по идее должны получаться окружности либо дуги?
353 468870
354 468877
>>68827
Нерва на делении основана же
355 468897
С какой скоростью надо пролететь над Ганимедом, чтобы нагреться об его тонкую атмосферу ?
356 468931
Почему ученые не пытаются создать сингулярности в лабораторных условиях?
357 468943
>>68897
с очень большой скоростью, на такой скорости тебе даже просто отдельные частицы из межпланетного пространства страшны.
358 468946
>>68931
Нет ускорителей такой мощности, да и даже если были — черные дыры подобного размера испарились бы моментально и изучать было бы нечего.

Более-менее стабильные черные дыры начинаются где-то от 1/1000 массы Земли.
359 469106
>>64488 (OP)
А вот возник вопрос, пока смотрел как дыры сливаются.

А как это происходит? Мгновенно или какое-то время занимает?
360 469107
>>69106
Пару лет
blackholemerger.jpg102 Кб, 527x615
361 469180
>>69106
Не мгновенно, но очень быстро, слева вверху же указано время. Начальное сближение может быть сколь угодно долгим, но в конце все происходит за доли секунды.

Чем ближе находятся тела, тем мощнее они излучают гравитационные волны, и тем быстрее они теряют энергию и падают друг на друга. Мощность излучения и скорость сближения пропорциональны кубу расстояния и очень быстро нарастают, в видео даже замедлить пришлось последние микросекунды.

В видео еще упрощенно показано искажение и соединение горизонтов событий, на самом деле оно тоже не мгновенно происходит. Там они должны быть не просто полусферической формы, а скорее в форме этаких запятых, где узкие части направлены друг на друга. Вершины удлинений касаются, сливаются, и перемычка быстро утолщается, пока черные дыры полностью не сольются в форму как на видео.
362 469203
>>69180
Если дыры могут слиться, возможен ли обратный процесс? Сингулярность разделяется на две части и образуются два горизонта событий? По CPT симметрии такое вроде должно быть возможным, не?
363 469207
>>69203

>По CPT симметрии такое вроде должно быть возможным


Зато второй закон термодинамики (а у ЧД охуенно огромная энтропия, причем энтропия итоговой дыры больше энтропии двух исходных) это запрещает for all practical purposes.
364 469279
>>66185

>был ли полет Бурана чем-то охуенным и выдающимся с технической точки зрения?


И был и не был.
Был - потому что первый сверхтяж на водороде, а СССР отставал в криогенных технологиях лет на двадцать, если не больше.
Не был - потому что для СССР этот проект был без реальных задач но с огромными затратами, прост наверху решили "сделойте такую же йобу, чтоб не стыдно перед буржуями".
Задачи Шаттла под которые выбивалось финансирование - обслуживание Хаббла и спутников шпионов сканировавших Атлантику, у СССР этих девайсов не было а сама экономика была из говна и палок.
365 469402
Почему нельзя просто выстреливать металлы и материалы на орбиту из пушки и уже там собирать всякие спутники и прочее.
366 469406
Анон, доставь вебку из какой-то серии "Expanse", там где полет молодого Манео через кольца Сатурна. Я знаю у тебя есть.
367 469407
>>69402

>Почему нельзя просто выстреливать металлы и материалы на орбиту из пушки и уже там собирать всякие спутники и прочее.



для этого на орбите придется построить завод, доменную печь, литейный цех и т. д.
368 469423
>>69402
1) Нужно пиздец какое ускорение, чтобы твой полезный груз запульнуть на орбиту, преодолев притяжение Земли и сопротивление атмосферы. Редкий груз такое вообще выдержит.
2) Нельзя вывести что-то на орбиту одномоментным ускорением. Или ты там его ловишь (компенсируя момент движком), или оно ебнется обратно на Землю. Перспектива так себе.
3) Как уже сказал >>69407, нужно будет создать практически полный производственный цикл на орбите. Мало того, что это стоит просто дохуя, кто все это будет обслуживать и поддерживать? На Земле, если у тебя ломается станок, приходит кузьмич, ну или приезжает на экскаваторе, и чинит. Кто на орбите будет ремонтировать?
4) Ну и плюс нужно разработать и отладить все технологии заново, учитывая отсутствие силы тяжести и отсутствие атмосферы (а значит, простого дыхания для обслуги, охлаждения конвекцией и источника кислорода для реакций горения). При том, что масштабные эксперименты в космосе дороги и небезопасны, потому что в случае проблем, оперативно устранять последствия затруднительно.
369 469464

> По его словам, уже «сейчас [генеральный директор госкорпорации] Дмитрий Рогозин дал указание начать разработку квантового двигателя».


>


> Бывший директор «Воронежского механического завода» (ВМЗ) Георгий Костин отметил, что в перспективе «квантовый двигатель» придет на смену жидкостному ракетному двигателю.


>


> Советник ракетно-космической корпорации «Энергия», министр общего машиностроения СССР (в 1983–1991 годах), Герой Социалистического Труда Олег Бакланов заявил, что «фундаментальная наука проникла в природу гравитации и антигравитации, квантованную структуру пространства. Это позволяет создавать нереактивные, не требующие химического топлива двигатели для космоса».


>


> По словам автора «квантового двигателя» Владимира Леонова, по космическому пространству «разлита» колоссальная энергия «в виде глобального электромагнитного поля с очень мелкой дискретностью (квантованностью), о котором ранее ничего не было известно». «Это глобальное поле открыто мной в 1996 году как пятая фундаментальная сила (суперсила) в виде сверхсильного электромагнитного взаимодействия (СЭВ). Его носителем является квант пространства-времени (квантон), размеры которого на десять порядков меньше атомного ядра, но он концентрирует энергию, намного превышающую ядерную», — говорит Леонов.


>


> Он заявляет, что «квантовый двигатель отталкивается от глобального поля СЭВ за счет деформации в нужном направлении (искривления по Эйнштейну) квантованного пространства-времени, создавая искусственною силу тяготения (тяги)».



Как тебе такое, Илон Маск?
370 469472
>>69464
Треды для щитпостинга:
>>468582 (OP)
>>469165 (OP)
371 469481
щас смотрел видео про космическую станцию и там вместе живут и мужчины и женщины.
Возник следующий вопрос: ебуться ли они между собой или дрочат? Они же там продолжительное время находятся, все здоровые и в сексуально-активном возрасте. Понятно, что работа у них сверх-ответственная и малейшая невнимательность может привести к плачевному исходу, но неужели они никак не выпускают пар? Что же там происходит?
372 469533
>>69472
Хорошо, вопрос такой: какие научные теории могут стать базисом для Роскосмоса при разработке квантового двигателя?
373 469539
>>69533
никакие, это бред сумасшедшего
более того, аналогичный антинаучный двигатель роскос уже запускал на орбиту
374 469547
>>69539
Хуясе. И как успехи? Ссылка есть?
375 469557
>>69547
ищи двигатель гравицапа если сильно интересно, он точно так неофициально назывался
сейчас все результаты поиска загажены ем-драйвом и прочим, мне впадлу серьезно искать
376 469586
Допустим, космический аппарат находится на орбите Солнца. Если включить ракетный двигатель выхлопом в сторону движения по орбите, и сбросить таким образом орбитальную скорость до 0, аппарат просто грохнется на Солнце?
377 469587
>>69586

> на орбите Солнца


вокруг Солнца
Фикс
378 469588
>>69586
Да. Правда, на солнечных орбитах (если не в районе Нептуна какого-нибудь) такие скорости, что это малореально.
379 469589
>>69588
Но были ли какие-то зонды, которые прямо на Солнце запуляли?
380 469590
>>69589
Сомневаюсь. Хотя что-то пробегало про какой-то исследовательский аппарат (то ли планируют, то ли уже запустили), который будет подлетать к Солнцу ближе, чем Меркурий. Но в любом случае это делается не так, как ты предлагаешь, а гравитационными маневрами. Пушо дешевле.
381 469593
>>69590
Окай, спс
382 469594
Допустим какой-то мегазлодей захотел спровоцировать солнце на мощный всплеск излучения с целью уничтожить жизнь на Земле. Как это теоретически можно сделать?
383 469595
>>69594
Постом выше выяснилось, что на Солнце даже не уронить ничего, даже если сильно захотеть, а ты про такое спрашиваешь))
384 469596
>>69595
А если облучать его сверхмощным рентгеновским лазером? Или выстрелить черной дырой?
385 469597
>>69590
Вот запустили https://en.m.wikipedia.org/wiki/Parker_Solar_Probe

> On 29 October 2018 at about 1:04 p.m. EDT, the spacecraft became the closest ever man-made object to the Sun


Значит на Солнце ничего еще не роняли
386 469598
>>69596
Как это теоретически можно сделать?))
387 469599
>>69598
С помощью гигантского лазера или ускорителя частиц разумеется.
388 469600
>>69595
Уронить можно, гравитационными маневрами. Другое дело, что даже если на Солнце уронить всю Землю, ему ровно нихуя не будет.
389 469602
>>69597
Ого, там не просто "ближе Меркурия", а в разы ближе. Ну, собственно, это подтверждает, что уронить можно: эта хуета уже не так и далеко от попадания в Солнце будет.
390 469605
>>69600

> даже если на Солнце уронить всю Землю, ему ровно нихуя не будет.


Оно взорвется.
391 469606
>>69605
Онемы пересмотрел?
392 469607
>>69602
Таки да
394 469694
>>69616
Вообще полезный спутник, со всех сторон.
395 469940
Что, если на первую ступень прикрутить вертолётные лопасти для посадки?
396 469942
>>69940
Не стоит.
397 469944
>>69942
А если всё-таки? Вдобавок не надо будет никуда возить, долетит своим ходом.
398 469946
>>69944
И все же не надо.
image.png257 Кб, 600x348
399 469948
>>69940
гугли rotary rocket
хуета
400 469949
>>69948
Они взлетать вроде хотели. А приземлятся можно на авторотации.
401 469959
Чем дальше объект, тем дольше идет свет от него, так? Мы видим эти объекты такими какими они были сотни, миллионы и даже миллиарды лет назад, так?
Почему тогда на самом горизонте видимой вселенной мы не наблюдаем момента зарождения вселенной?
402 469961
>>69959

>Чем дальше объект, тем дольше идет свет от него, так? Мы видим эти объекты такими какими они были сотни, миллионы и даже миллиарды лет назад, так?


Да.

>Почему тогда на самом горизонте видимой вселенной мы не наблюдаем момента зарождения вселенной?


Потому что изначально Вселенная была непрозрачна для света: слишком много слишком горячего вещества. Плюс, то что мы видим, видим мы хуево. Во-первых, расстояние пиздец огромное: различить объекты мельче галактик (причем ярких) вообще проблематично. Во-вторых, для далеких объектов красное смещение становится очень большим.
403 469962
>>69959
Потому что сначала вселенная была непрозрачной. Самое раннее, что можно увидеть - реликтовое излучение, которое появилось через 400000 лет после БВ.
404 469964
Почему в нашем ближайшем нет квазаров - до всех миллиарды светолет? Со временем они успокаиваются, и поэтому в современной Вселенной их почти нет? Или "на отдалении" они тоже редко встречаются и от одного до другого обычно очень далеко?
405 469969
>>69964
Потухли жи, те что далеко тоже на данный момент потухли, но мы их до сих пор видим светящимися из-за конечности скорости света.
будущее россии.jpg52 Кб, 612x546
406 469976
>>69949

>А приземлятся

1317449868797.jpg181 Кб, 600x415
407 469977
>>69464
Это откуда, ссылки то блять где? Уёбище ты лесное.
даблфейспалм1.jpg44 Кб, 600x530
408 469978
>>69616
По началу вроде ничего особенного. Но когда читаешь раздел "Критика", становится понятно, что за "академики" и "инженеры" возглавляют российскую науку. Пиздец... просто пиздец...
409 469984
Что космонавт увидит в центре черной дыры; если дыра большая его же сразу не порвёт, когда он пройдёт через горизонт событий.
410 469993
>>69984
Ничего не увидит, в черной дыре невозможно получить какую-либо информацию с большей глубины. Все направления ведут только глубже вниз, к сингулярности, у света нет возможности как-то вернуться назад и донести тебе информацию о том, что находится ниже.

Да и вообще никакого зрелища не будет, если космонавт свободно падает в сверхмассивную черную дыру, то он даже не заметит собственно момент пересечения горизонта. Вплоть до самой спагеттификации и разборки на атомы ему будет казаться, что горизонт событий все еще находится на каком-то расстоянии перед ним, и что чернота горизонта заполняет менее половины неба.

По ссылке https://jila.colorado.edu/~ajsh/insidebh/schw.html есть хорошее объяснение и видео с симуляцией.

https://vimeo.com/8723702
https://vimeo.com/8818891
411 469995
Как известно вращение Земли замедляется и Луна отдаляется. Если расчеты с какой скорость вращалась Земля, после того как они более мене сформировались с Луной?
412 469999
>>69995
Точно не известно, но обычно считается, что часов 5-6.
14778204099140.jpg44 Кб, 500x500
413 470040
А скажите ка мне, вот подтвердили наличие гравитационных волн.
Гравитон безмассовая частица со спином, имеет поляризацию.
Можно построить квантовый усилитель гравитационных волн тогда, типа лазера?

ГРАЗЕР?

Можно ведь за счет резонанса усилить тогда гравиволны одной поляризации, даже совсем слабые, от слабых источников и ИРЛ наблюдать/изучать тогда гравитацию?
Вот только не ясно, что может быть в качестве "поляризационоого фильтра" для гравиволн.

Как вам идея, ИЛОНЫ МАСКИ, а?

Такой ГРАЗЕР может локально деформировать пространственно-временной континуум в выбранном направлении например и служить мощным оружием и/или движителем в пространстве,так ведь?
image.png462 Кб, 950x506
415 470042
>>70040

>ГРАЗЕР может локально деформировать пространственно-временной континуум в выбранном направлении например и служить мощным оружием



Дратути
416 470093
Наверняка же можно рассчитывать на то, что когда следующий метеорит с Марса упадёт на Землю, НАСА сможет найти место, откуда он откололся на Марсе, найдя новый кратер сравнением снимков с реканессенс орбитера, так? Это должен быть приличных размеров кратер, чтобы кусок Марса улетел с него.
417 470094
>>70093
Более того, если расставить на Марсе регистраторы землетрясений, то сопоставить время и найти место будет еще проще. Хотяя, я ща подумал, что может много времени пройти с момента откалывания куска Марса до момента его падения на Землю
418 470095
Если завтра на Землю ебнется метеорит, аналогичный по размеру и скорости тому, что вызвал мел-палеогеновое вымирание, какие будут последствия для людишек? Скажем, ебнется в густонаселенном районе прямо на Нью-Йорк, рряяяя. Интересуют и локальные и глобальные последствия.
419 470098
>>70040
Подзагугли хотя бы маняпредположения о длине гв.
После этого предположи кратные размеры резонатора. Даже не вдаваясь, из какого маняматериала он должен быть изготовлен.
420 470099
>>70095
Во-первых, мы о нём, скорее всего узнаем заранее. Челябинский метеорит мы пропустили потому что он был маленький и двигался со стороны солнца. Так что в густонаселённом районе не упадёт точно. Если мы узнаем о нём хотя бы за месяц, то людей эвакуируют, а если хотя бы за несколько дней (а меньше не может быть для такой глыбы), то эвакуируют хотя бы большинство из самых ближних районов (ну и зависит от густонаселённости конечно, в логистику вдаваться лень). Дальше будут прямые последствия, разрушения, цунами, вот это всё, но это всё будет известно заранее, людей, опять же, везде эвакуировали. А вот косвенные последствия гораздо существеннее. Скорее всего похолодает, небо будет закрыто выброшенным пеплом, это сильно скажется на урожайности и на экосистемах по всему миру. Переселение огромного количества людей тоже дело не из приятных, и экономика на такое не рассчитана. Ну так, если примерно почувствовать, то от прямых последствий погибнет ну может быть пара сотен тысяч человек, а может и гораздо меньше, а от косвенных сотни тысяч или даже миллионы. Ну а так, в остальном похуй вообще, цивилизация не пострадает, оправимся лет за 50.
421 470100
>>70094

>может много времени пройти с момента откалывания куска Марса до момента его падения на Землю


Я больше скажу, вот так вот чтобы откололся и сразу прилетел - это крайне маловероятно.
422 470101
>>70099
Где-то (по Дискавери?) еще показывали, что удар такой силы выбросит осколки в ближний космос, которые потом, естественно, упадут обратно. И будет "мелкий метеоритный дождь" по всей планете. Или пиздели?
423 470102
>>70098

Подзагуглил. 10^11 Гц максимум. Так что все ок. Конструкция приемлимых размеров вполне возможна.
424 470103
>>70101
Ну это да, это возможно.
425 470109
>>70099
А разве нельзя об него ебнуть какой-нибудь искусственный спутник или ракету?
426 470114
>>70109
А толку, что ты сделаешь глыбе космического говна весом в хуиллиард тонн?
427 470115
>>70114
Ядерной бимбой расколоть можно
428 470116
>>70114
Траекторию чуть поменять.
429 470119
>>70115
>>70116
Нельзя. Он наши бомбы вообще не заметит. А если и удастся расколоть, то ещё может стать только хуже. Вместо одного предсказуемого камня с хорошо измеренной траекторией будут несколько, всё ещё гигантских, летящих в разные места планеты, плюс ещё куча мелких, и точная траектория их всех будет неизвестна.
430 470120
>>70119
Зависит от размера наверное, если вместо одного такого, какой атмосфера не сотрет, будет 5 которые сотрутся об нее - будет гораздо лучше. А бимбу можно любой мощности сделать, можно и Луну к херам взорвать если захотеть, наверное
431 470123
>>70120

>Зависит от размера наверное


Анон изначально спрашивал про такой, как 65 миллионов лет назад упал, он был около 10 км в диаметре, о таком и речь.

>если вместо одного такого, какой атмосфера не сотрет, будет 5 которые сотрутся об нее - будет гораздо лучше


Да, но такой большой астероид, как мы рассматриваем, даже если расколоть на 5 частей, это всё ещё будут огромные глыбы, которые легко пройдут атмосферу.

>А бимбу можно любой мощности сделать, можно и Луну к херам взорвать если захотеть, наверное


Эм, нет, это абсолютно за гранью наших возможностей. Какую там Луну, о чём ты. Мало того, что бомбу надо сделать, её ещё туда доставить надо. Ну а бомбу чтобы взорвать Луну мы ещё в ближайшие столетия не сможем создать как минимум.
432 470124
>>70123

> это абсолютно за гранью наших возможностей


А я читал, что ядерные бимбы году к 70му СССР и США научились делать практически неограниченной мощности

А доставить на Луну ракетой-носителем можно, как и обычные ядерные бимбы токо покруче
433 470131
>>70124
Бля, чувак, ну что значит неограниченной мощности? Мощность ограничена количеством вещества, которое вступает в реакцию. Да, ты можешь сделать бомбу любой мощности, если соберёшь достаточно дейтерия, лития, урана, что там ещё в них используется. Чтобы взорвать Луну не хватит всех запасов, которые у нас есть, всех бомб, которые когда-либо были созданы. Всего современного ядерного арсенала возможно не хватит, чтобы расколоть даже 10-километровый камушек. Ну какая Луна, ты хоть представляешь себе её масштабы? Бомба для взрыва Луны должна быть сама несколько километров в диаметре.

>А доставить на Луну ракетой-носителем можно, как и обычные ядерные бимбы токо покруче


Ну да блядь, делов-то, всего лишь ракету сделать. Ну так, на несколько миллионов тонн полезной нагрузки. Муск в гараже состряпает.
434 470135
>>70131

> Всего современного ядерного арсенала возможно не хватит, чтобы расколоть даже 10-километровый камушек.


Вот уж в этом сомневаюсь, бомба раз в 100 мощнее царь-бомбы наверняка ее расколет
435 470138
>>70135
Может быть. Но доставить её туда не представляется возможным. Ну и, как я уже говорил, это мало чем поможет, а то и хуже сделает.
15519006192580.png294 Кб, 692x395
436 470139
>>70138

> Но доставить её туда не представляется возможным.

437 470155
>>70135
>>70138
Ловите пасту.

> Ну давай прикинем порядок. Энергия гравитационной связи Земли (https://en.wikipedia.org/wiki/Gravitational_binding_energy) около 2 10^32 Дж. Это та энергия, которая нужна чтобы разорвать планету на части. Будем считать, что чтобы она треснула, достаточно 1/50 части этой энергии (хотя этого наверняка слишком мало), то есть 4 10^30 Дж.


>


> Тонна в тротиловом эквиваленте примерно равна 4 10^9 Дж. То есть, нам понадобится около 10^21 тонн в тротиловом эквиваленте.


>


> Царь-бомба имела массу порядка 25 тонн и выдала мощность порядка 50 Мт. То есть, для термоядерного заряда примерная удельная мощность - 2 10^6 тонн в эквиваленте на тонну заряда. Таким образом, гипотетическая термоядерная бомба, способная расколоть Землю, будет весить примерно 10^21 / 2 10^6 = 5 10^14 тонн.


>


> Масса Земли - 6 10^21 тонн. У нашей бомбы - примерно в 10^7 раз меньше. Значит, если сделать бомбу в виде шара с плотностью как у Земли, то он будет в 200-250 раз (кубический корень) меньше Земли радиусом, то есть с радиусом в 25-30 км.


>


> Чтобы расколоть Землю понадобится термоядерная бомба в виде шара диаметром примерно 50-60 км.



В 100 раз мощнее Царь-бомбы - курам на смех. Собственно, метеорит, упавший 65 миллионов лет назад был в 2 миллиона раз мощнее той бомбы. И хули, он даже тоненькую (менее процента радиуса) земную кору не пробил.
438 470160
>>70155
50 км шар - ого. Я разочаровался в ядерных бомбах;(

На Землю диаметром 12000 км бомба диаметром 50 км - в 240 раз меньше

Но если тот метеорит был 10 км, то бомба, которая его расколет, будет по аналогии 10 / 240 = 41 метр диаметром. Царь-бомба была 8х2 метра. Нарастить раз в 6 - и рванет что надо.
439 470163
>>70160
Ну там гораздо меньшего хватит, потому что метеорит сам по себе имеет крошечное тяготение. То есть энергия гравитационной связи падает/растет гораздо быстрее, чем куб радиуса (не скажу с какой именно скоростью, правда). Но для Луны тяготение уже значительно.
440 470165
>>70160
Не в 6, а в ~1000
441 470166
>>70163
О, значит метеорит вообще на дохера кусков разлетится и все будет чики-пуки
442 470167
>>70165
41х41= 1681
8х2=16
В сто раз, в точности как я и говорил изначально)))
А учитывая слабую гравитационную связь и того меньше
443 470168
>>70167
А, падажжи, это ж куб, тогда да, хировато
444 470175
В любом случае, мы даже царь-бомбу не сможем дальше НОО отправить с существующими ракетами.
445 470177
А ведь еще можно лазером что-нибудь сделать с астероидом - испарять на одной его стороне что-нибудь (газы чтоб вылетали реактивной струёй) и таким образом отклонить орбиту, или бред несу?
446 470178
>>70177
Если у тебя вагон времени, то можно, в теории. На практике - сам понимаешь.
447 470179
О, статейку надо потом почитать как все на самом деле https://en.m.wikipedia.org/wiki/Asteroid_impact_avoidance
448 470194
>>70163

>энергия гравитационной связи падает/растет гораздо быстрее, чем куб радиуса (не скажу с какой именно скоростью, правда)


А, собственно, в той статье есть формула. Она для равномерной плотности, но это не принципиально для порядка, там только коэффициент будет другой. Короче, учитывая, что масса растет как куб радиуса, получаем, что энергия гравитационной связи пропорциональна пятой степени радиуса. Так что для астероида 10 км в диаметре хватит и одной царь-бомбы за глаза.
449 470199
>>70155

>Таким образом, гипотетическая термоядерная бомба, способная расколоть Землю, будет весить примерно 10^21 / 2 10^6 = 5 10^14 тонн


Бля ну охуеть теперь экстраполяция.жпг
450 470210
Почему на первой ступени ракеты-носителя не использовать обычный ЖРД с окислением атмосферным воздухом? Нахуя на земле сжигать драгоценнейший жидкий кислород из баков? Разве так мы не сэкономим на массе? На атмосферном участке полета нам тяга не так важна: тут задача лишь в том чтобы выйти в стратосферу, а там уж горизонтально будем разгоняться как положено.
451 470217
Ух блин, посмотрите видос (под первой картинкой) - скоко проблем-то с сексом в космосе! https://en.wikipedia.org/wiki/Sex_in_space
452 470218
Как принять участие в лунном заговоре?
453 470237
>>70218
Родиться пиндосом в 20-40-е годы.
454 470245
Меня очень сильно пугают чёрные дыры, да и вообще осознание того, что мы живём на шаре который летит хуй знает куда. Помню один раз накатил SpaceEngine и сам безграничный космос меня просто пугал похлеще любого хоррора. В Интерстелларе, когда показывали чёрную дыру, мне просто становилось не по себе. Я один тут такой ебанутый?
455 470246
>>70245
Тут когда-то был целый тред таких ебанутых, не знаю, что с ним сейчас, надеюсь потёрли.
image.jpeg1,1 Мб, 2208x1242
456 470255
Вот еще про бимбы и астероиды
457 470263
>>70210

Не используют, потому что таких долбоебов нет.

Во-первых, любые воздушно-реактивные двигатели обладают ничтожной удельной тягой по сравнению с ракетными, в десятки раз ниже. Соответственно, чтобы ракета имела тяговооруженность больше 1 и вообще могла хотя бы подняться со стартового стола, устанавливаемые на нее двигатели уже получаются размером с саму ракету.

Во-вторых, любой воздушно-реактивный двигатель максимально эффективен только на малом диапазоне скоростей. Турбореактивные хорошо работают в начале, но начинают глохнуть уже после двух скоростей звука, прямоточные ВРД лучше работают на нескольких скоростях звука, но стоя на месте не выдают вообще никакой тяги и для старта с земли абсолютно непригодны, выше 5 махов нужно сверхзвуковое горение, с которым пока сильные проблемы и тоже нужен свой двигатель.

В-третьих, воздух при подъеме очень быстро заканчивается. Первая ступень того же фалкона сбрасывается на высоте 60+ километров и скорости ~5 махов, на такой высоте ни один современный двигатель не работает.

Так что либо ты изобретаешь какие-то сказочные двигатели, которые одновременно и с места могут выдавать невиданную тягу, и на гиперзвуке работать, и весить немного, либо твоя ракета получается увешанной целыми батареями разнорежимных двигателей, либо ты не занимаешься долбоебизмом и окислитель с собой возишь, как все нормальные люди.
458 470401
>>70160
50км - это размер заряда.
Размер заряда кузькиной мамки был ~0,5-1,0м.
К 50км заряду добавь ещё всю инициирующую обвязку. При таких масштабах она легко приблизится к размерам Луняши.
459 470415
>>70401
Там при расчете бралась полная масса матери, с инициируещей обвязкой аэродинамическими конструкциями и т. д. В оригинале вроде даже был свинцовый балласт как замена урану, который бы добавил мощи. Так что норм. Хотя боньба таких размеров потребовала бы еще всякой несущей инженерной хуйни будь здоров, это да.
460 470448
>>70175
Запулить бомбу, потом разгонную ступень для неё отдельно, состыковать. Пиздец вы тут маньки.
461 470462
>>70448
Это в теории (или в Кербале) все возможно. На практике нужны годы разработок, чтобы не получился очередной роскосмос на орбите.
462 470518
Что если темная материя это триллиарды черных дыр, которых мы не видим только из-за того что в данный момент времени ничто не взаимодействует с ними?
464 470520
>>70518
Гугли планковские черные дыры. Короче, есть такая версия. Только там не "в данный момент", а вероятность с ними провзаимодействовать вообще ничтожная, меньше чем с нейтрино.
465 470535
>>70520
Ну, темная материя же влияет на расширение, планковские черные дыры не способны на такое?
Я имею ввиду что если допустим все межзвездное пространство заполнено черными дырами, а звезды образовались только потому что рядом таких скоплений не было, а современные звезды с ними нашли равновесие за столь долгое время. И допустим, пошлем мы вояджер за пределы солнечной системы, а тот бац и упадет в черную дыру. И так по всем направлениям, и мы не сможем улететь из нашей вселенной.
14087360461270.jpg120 Кб, 662x900
466 470553
Если в космосе окажется что-то вроде ракеты союз полностью заправленной, куда можно закинуть аппарат весом например в 3 тонны за 10 лет полета?
467 470573
>>70518
Ну это один из теоретических вариантов, подразумевающий барионный ориджин темной материи. Там не только черные дыры, но много помимо них вполне привычной формы вещества в кандидаты. Просто по уравнениям эволюции вселенной не сходится.
468 470575
>>70535

>Ну, темная материя же влияет на расширение


На расширение влияет темная энергия. Это "еще более темная" хуйня. По теориям (тут ОТО ни при чем), вакуум или пространство само по себе обладает таким свойством.

>Я имею ввиду что если допустим все межзвездное пространство заполнено черными дырами, а звезды образовались только потому что рядом таких скоплений не было, а современные звезды с ними нашли равновесие за столь долгое время. И допустим, пошлем мы вояджер за пределы солнечной системы, а тот бац и упадет в черную дыру. И так по всем направлениям, и мы не сможем улететь из нашей вселенной.


Массивных черных дыр не может быть так много, это не увязывается с наблюдениями.

Планковские черные дыры, как раз, имеют малую массу - планковскую (~20 миллионных грамма). А поскольку гравитация вообще-то слабая штука (тебя сильно притягивает табуретка?), то такая дыра тянет очень-очень слабо, да и радиус у нее милипиздрический. Такая дыра сквозь Солнце пролетит и ни с чем не прореагирует.
469 470592
>>70575

>тебя сильно притягивает табуретка?


Ускорение свободного падения зависит не только от массы, но и от плотности, поэтому аналогия с табуреткой некорректна. Сжатая до планковских размеров табуретка будет тянуть сильнее.
Другое дело, что у микрочёрной дыры малое сечение захвата и расталкивающее падающее вещество давление излучения хокинга, да и срок жизни у них короткий.
470 470593
>>70592

>Ускорение свободного падения зависит не только от массы, но и от плотности


Нет, не зависит. Посмотри на закон всемирного тяготения: сила зависит только от массы притягивающего вещества и от растояния. Все.
471 470606
>>70593

>Посмотри на закон всемирного тяготения


Нет, извини, в нашем треде смотрят больше на уравнения Эйнштейна.
472 470607
>>70263

>воздушно-реактивные двигатели обладают ничтожной удельной тягой


Нихуя себе. По ходу дела сверхзвуковых стратосферных самолетов не существует, это газетная утка, а миг-25 просто троллинг и фейк.

>устанавливаемые на нее двигатели уже получаются размером с саму ракету


Уравнения приведи, а не сопливые вскукареки(хуй знает, как ты считаешь размер движка?)
изображение.png301 Кб, 640x480
473 470608
>>69948
блядь
474 470610
>>70593
Очевидно, что >>70592-анон имел в виду g на поверхности тела. А оно растёт при фиксированой массе тела и увеличении его плотности, просто потому что растояние от поверхности до центра падает.

>>70606
А? Я ненастоящий сварщик, но вроде именно в этом вопросе у Эйнштейна то же, что у Ньютона.
475 470612
>>64488 (OP)
В центре Земли находится частичка чёрной дыры, такой же как в центре галактики. Образовалась материя в чёрной дыре - в результате взрыва сверхновой галактики. Влияние этой материи на материю объективного мира с нашими уровнём развития органов осязания и объективного понимания - воспринимается как чёрная дыра. Хотя ни дырой и ни чёрной она не является.
Где я что не правильно понял?
476 470613
>>70612

>Где я что не правильно понял?


Я бля даже не знаю, что тут ответить... Ты где всю эту наркоманию вообще взял?
477 470614
>>70607
Двигатель Д-30Ф6, устанавливаемый на МиГ-31 — масса 2416 кг, максимальная тяга на форсаже 152 кН, удельная тяга 6,41.
Мерлин-1Д, устанавливаемый на Фалкон 9 — масса 470 кг, максимальная тяга 845 кН, удельная тяга 180.
478 470615
>>70613
Ладно - как ты объяснишь наличие оче оче высокой температуры в центе Земли? Как у Солнца - термоядерный взрыв водорода?
sage 479 470616
>>70615

>наличие оче оче высокой температуры в центе Земли


Гравитационное сжатие и постепенный распад радиоактивных элементов, входивших в состав протопланетного облака, из которого появилась солнечная система. Эти радиоактивные элементы все тяжёлые и поэтому утонули в ядро Земли. (В первую очередь, наверное, уран-235? Тут я хз.)

>у Солнца - термоядерный взрыв водорода


Не взрыв, а относительно спокойное горение.
480 470617
>>70615
Нет, термоядерных реакций там нет, планета с самого начала была горячей за счёт самого процесса планетообразования и перемешивания внутренностей, и затем начала остывать, начиная с внешних слоёв, и всё ещё остывает. Луна несколько замедляет остывание своими приливными силами.
481 470620
>>70616

>протопланетного облака


почему внутри планеты земля огненный шар?
ты понимаешь какие температуры должны быть чтобы двигать континенты и вулканы ебашить...
>>70617

>и затем начала остывать


чтобы так долго остывать вещества в планете нужно в тысячу раз больше по объёму.
я напомню планете 5 лярдов лет и вулканы до сих пор хуярят только в путь.
482 470629
>>70620

>почему внутри планеты земля огненный шар?


Не огненный шар, а раскалённое железо.

>чтобы так долго остывать вещества в планете нужно в тысячу раз больше по объёму.


Расчёты-то выложи свои, а то мне интересно, откуда ты вообще исходные данные брал.
483 470631
>>70616

>уран-235


Уран 238 тяжелее, чем 235, а значит в центре Земли был бы именно он. Но 238 не распадается, а значит Земля внутри не была бы горячей.
Шах и мат, шаросектанты.
484 470634
>>70620

>чтобы так долго остывать вещества в планете нужно в тысячу раз больше по объёму.


Ты примерно почувствовал?
485 470667
>>70634
Прошлый год: 674 тысячных. Этот год: 704 тысячных. Да, извини, совсем другая картина, что это я.
486 470668
>>70667
Простите, поцоны, промазол.
487 470674
>>70614
А теперь с массой окислителя, плез.
488 470675
>>70674
Турбина будет работать до 20 км, и всё. Ракета до туда долетает за секунды.
489 470676
>>70675
А до сюда?
490 470679
>>70676
До сюда минут за 20 после обнаружения, до метро добежать успеешь.
491 470690
>>70679
Если это не питерское метро то можно не потеть перед смертью.
492 470762
>>70612
Пиздец. Это нечто гениальное. Анон, ты не из будущего к нам пришёл?
493 470931
Если объект находится на орбите планеты и достигает второй космической (и превышает её), то он в любом случае покинет орбиту этой планеты?
494 470938
>>70931
Да, при условии что он разгоняется прямо по курсу, ну или куда-то ещё, но не в планету или её атмосферу.
495 470941
>>70938
А на орбите на скорости более, чем вторая, как можно остаться? Только корректируя курс двигателями (говорим о космическом корабле)?
496 470945
>>70941
Ага. Нужно будет постоянно жарить двигателями в направлении планеты (или строже говоря, барицентра), имитируя таким образом более сильное гравитационное притяжение.
497 470946
>>70945
Спасибо.
498 470955
>>70941
Ну, ещё можно привязаться верёвочкой, или удерживаться магнитным полем (частицы в радиационных поясах так и делают).
500 471015
>>71006
планктон
501 471016
>>71015
Так сильно что аж засветку создает?
502 471029
>>71016
на таймлапсе небось
squidfishingboat[1].jpg68 Кб, 720x480
503 471032
>>71006
>>71015
Никакой это не планктон, это рыбацкие суда прожекторами приманивают улов.

https://earthobservatory.nasa.gov/features/Malvinas
504 471043
>>71032
Какой же улов тупой, пиздец просто.
image.jpeg64 Кб, 365x228
505 471057
>>64488 (OP)
Если можно видеть галактику, изображение которой шло до земли 10 миллиардов световых лет, значит можно посмотреть «в спину» этому изображению, или я что-то не так понимайу
506 471079
>>71057
Что значит "в спину"? Ну, ты видишь галактику такой, какой она была 10 млрд. лет назад, такая себе бесплатная машина времени над головой.
507 471088
>>71057

>значит


Не улавливаю логику.
508 471118
Почему лед с мелких комет и астероидов не сублимируется и не улетает в космос? У них же нет гравитации, вода должна свободно улетать, разве нет?
509 471141
>>70620
Пиздец дегенерат. И бывают же такие.
510 471149
>>71057
Если ты имеешь в виду посмотреть в противоположную галактике сторону и увидеть как она выглядит с другой стороны то это не возможжно. свет удаляется со световой скоростью, и догнать своими глазками ты его никак не сможешь..
511 471191
>>71118
С чего бы ему сублимироваться, когда он холодный и твердый? При нагреве сублимируется, если тело вблизи Солнца пролетает, а без нагрева лед стабилен.

>У них же нет гравитации


Гравитация есть у всего. Как, по-твоему, эти самые кометы и астероиды образовались изначально?
potw1208a.jpg58 Кб, 700x664
512 471209
У меня тупой вопрос.

Когда Эта Киля взорвется как гиперновая, то это как-то повлияет на Землю? Грубо говорят до нас "достанет" гамма?
513 471222
>>71209
Если ось вращения не направлена на Землю, то ничего особенного не будет. Если направлена - то пиздей. Вычитал в Википедии: https://ru.wikipedia.org/wiki/Эта_Киля#Возможное_влияние_на_Землю
514 471240
>>71222

>Если направлена - то пиздей



Нас сожжет насмерть?
515 471250
>>71240

> ...прямое попадание гамма-всплеска может привести к катастрофическим повреждениям и серьезному массовому вымиранию. Расчеты показывают, что накопленная энергия такого гамма-всплеска, поразившего земную атмосферу, будет эквивалентна одной килотонне тринитротолуола на квадратный километр по всему полушарию, обращенному к звезде, причем ионизирующее излучение будет в десять раз превышать смертельную дозу облучения всего организма[114].



Оттуда же.
516 471254
>>71250
А в другом полушарии будет похуй?
517 471267
>>71254
Не совсем. Такой пиздец приведет к глобальному изменению климата. Первые дни будут ураганные ветры по всей планете, как я понимаю. Возможно, атмосферу вообще сильно проколбасит, но с другой стороны химсостав поменяться не должен (ну озон там распадется, будет хорошо жарить ультрафиолетом сколько-то там лет). Но по сравнению с "затронутым полушарием" да, легко отделаются.

Алсо, там последствия неравномерны для полушария неудачников. Величина гамма-облучения будет убывать как косинус угла источника в небе. У кого он будет 90 градусов (зенит), те прожарятся особенно хорошо.
518 471270
>>71254
Смотри последствия всяких пермских или силлурийских вымираний.
519 471275
>>71250
Так одна килотонна на квадратный километр это же вообще ничто, на квадратный метр это всего-то 1 грамм тротила или 4200 джоулей, Солнце в зените столько заливает за четыре секунды. Гамма-всплеск тоже не моментально происходит, так что с чисто энергетической точки зрения эффект будет тот же, как если бы на несколько секунд зажглось и погасло второе Солнце.

Стоящие прямо под лучами возможно и подохнут от радиации, да, но людям под землей или на другой стороне планеты должно быть полностью похуй.
520 471278
>>71275

>одна килотонна на квадратный километр это же вообще ничто, на квадратный метр это всего-то 1 грамм тротила или 4200 джоулей


Три нолика потерял. Это один килограмм на квадрат.

Можно прикинуть через площадь Земли. У меня получилось 250 000 мегатонн в эквиваленте. Впрочем да, ничего такого ужасного. Для сравнения, чиксулубский метеорит оценивается в 100 000 000 тератонн, в 400 раз больше.
521 471279
>>71278

> 100 000 000 мегатонн


быстрофикс
106067183fireballchart6402x-nc.png152 Кб, 1335x1059
522 471312
Че за дичь?

В декабре 2018 года произошёл крупнейший взрыв метеорита после случая в Челябинске в 2013 году.

Метеорит при разрушении об атмосферу выделил энергии в 10 раз больше, чем атомная бомба, сброшенная на Хиросиму. Однако это произошло вдали от людей, поэтому явление осталось практически незамеченным. Об этом рассказал «Би-би-си» сотрудник планетарной защиты NASA Линдли Джонсон.

https://tjournal.ru/science/90583-nasa-rasskazalo-o-krupneyshem-posle-chelyabinska-vzryve-meteorita-na-kamchatke-kotoryy-nikto-ne-zametil
https://www.bbc.com/news/science-environment-47607696

Я могу поверить, что кроме швятых сотрудников НАСА никто не наблюдает ни за космосом, ни за атмосферными явлениями, но чтобы "никто не заметил взрыва мощностью 10 Хиросим"? Современные средства обнаружения ракетных пусков позволяют фиксировать старт отдельных ракет по огневому следу, чуть ли не крылатые ракеты уже видят, а метео спутники фиксируют изменения температур в пределах десятых долей одного градуса. ЧТО ЗА ДИЧЬ?
523 471324
Неужели Вселенная такая маленькая и виде тора??
525 471327
За пределами Метагалактики, возможны ли объекты(в виде других Метагалактик или других структур) которые произошли не от Большого взрыва?
526 471335
>>71327
Хуй знает.
527 471351
>>71327
Один хуй. Если они и есть, мы с ними никак не взаимодействуем. Ну или так своеобразно взаимодействуем, что все эти взаимодействия замечательно описываются внутри нашего мира.
528 471364
Ещё тупых вопросов.
1. Тело выведенное на эллиптическую орбиту со временем там и останется, или орбита будет стремиться к окружности?
2. Пикрил с красной стрелкой. Если телу на орбите (чёрная) будет предана небольшая скорость в направлении Земли (синий круг) (например, исчисляемая всего десятками метров в секунду), то это тело в конце концов должно же упасть на Землю?
3. А если телу предать скорость в противоположном напревлении от п.2, то тело в какой-то момент придёт к относительно стабильной высоте за счёт гравитации земли?
4. А если как в п.3, но телу на орбите придать не просто скорости, а, например, в десятки раз больше, чем вторая космическая для этой планеты? Как тело уйдёт с орбиты? Ближе к зелёной траектории, или всё же планета будет оказывать и тут существенное влияние и оранжевая траектория ближе к правде?
5. На орбите ориентация космических аппаратов параллельно земной поверхности отсутствует же и остаётся относительно постоянной?
529 471368
>>71364
1. На очень больших промежутках времени будет стремиться к окружности, на масштабах тысяч лет и менее — можно считать, что останется такой же, как и была, если тяготение других тел не сместит орбиту.

2 и 3. Ничего никуда не упадет и не улетит, просто чуть-чуть поднимется или опустится перицентр/апоцентр орбиты, будет у тебя орбита не 400×400 км, а, скажем, 390×400 или 400×410.

4. Если скорость именно в десятки раз больше — практически по прямой линии, тело улетит слишком быстро, чтобы планета могла существенно изменить траекторию.

5. Отсутствует. Если хочешь, чтобы одна сторона спутника всегда смотрела на Землю — придавай ему вращение со скоростью 1 оборот вокруг своей оси за 1 виток орбиты. МКС, например, так и вращается.
530 471370
Это правда, что настоящий спейсачер всегда выберет первый вариант?
531 471375
>>71267
Какой ужас. Как спастись от гамма-вспышки?
532 471378
>>71364
1. Если не учитывать влияние сторонних тел, приливных сил и всяких тонких эффектов ОТО (то есть, если взять сферическое тело в вакууме), то останется навсегда.
2. Не понял, ты постоянно собираешься жарить в сторону Земли или однократно? Если однократно - перейдет на другую орбиту, но проходящую через точку, где придали ускорение. Если постоянно - то рано или поздно ебнется, но по-моему даже выйдет за первоначальный круг.
3. Аналогично для однократного, для постоянного - улетит нахуй.
4. Оранжевая ближе, но все зависит от точного значения скорости.
5. Она постоянная "относительно звезд". Если хочешь постоянно быть повернутым к Земле пузом, нужно вращаться с той же скоростью, что и круги мотаешь.
533 471379
>>71351
тёмная энергия например
534 471380
>>71378

>но по-моему даже выйдет за первоначальный круг


Ща подумол, пожалуй, не выйдет. Просто будет снижаться, причем все быстрее и быстрее, пока не ебнется.
536 471434
>>71312
А кто этот взрыв непосредственно мог напрямую наблюдать? Два с половиной рыбака, вот и все. А соответствующие службы наблюдают.
537 471524
>>71434
>>71312
Ну так а хули службы молчат? В службах роботы работают или десятки тысяч живых людей?
https://www.youtube.com/watch?v=bQy30M9P6cE
538 471525
>>71370
Нет, сникерс нажористее
539 471756
>>64488 (OP)
Что за срань, эти ваши страпельки?
540 471845
Ещё вопрос по орбите. Если орбита спутника наклонена относительно оси Земли, при этом направление движения спутника совпадает с направлением вращения Земли, то что случится, если начать корректировать курс спутника, увеличивая наклон и пройдя таким образом в какой-то момент ось вращения Земли? Спутник будет двигаться в противоположную от направления вращения земли сторону?
541 473657
Возможно ли существование целого кольца на орбите планеты? Не кольца из камней, как у Сатурна, а целого монолитного вращающегося кольца.
Примерно чувствую, что такое кольцо существовать не сможет, но не могу объяснить почему:

В принципе, целое монолитное кольцо можно представить состоящим из отдельных близких кусков, летящих по орбите, и тогда кольцо должно быть стабильным.

Еще можно представить, что мы берем кольцо как у сатурна, и начинаем протягивать между отдельными глыбами тонкие нити. На этом этапе ничего не должно произойти. Начинаем постепенно утолщать нити, пока помностью не заполним промежутки между камнями. В какой момент все это обрушится?
542 483223
>>73657
Угловая скорость вращения тем больше, чем ближе к планете. Если кольцо Сатурна внезапно сделать целым - его разорвет с внутренней стороны гравитацией, с внешней - центробежной силой, прочность на таких масштабах ни на что не влияет. Очень тонкое кольцо сможет просуществовать некоторое время, но будет неустойчивым по тем же причинам.
Тред утонул или удален.
Это копия, сохраненная 28 июля 2019 года.

Скачать тред: только с превью, с превью и прикрепленными файлами.
Второй вариант может долго скачиваться. Файлы будут только в живых или недавно утонувших тредах. Подробнее

Если вам полезен архив М.Двача, пожертвуйте на оплату сервера.
« /spc/В начало тредаВеб-версияНастройки
/a//b//mu//s//vg/Все доски